Remembrances Flashcards
What prevents polyspermy?
- Capacitation
- Cortical reaction
- Acrosome reaction
Cortical reaction
(In the regulatory zone) What is the upstream DNA sequence that leads to increased transcription? (2021, 2022)
- Coactivator
- Coregulator
- Enhancer
- Inhibitor
Enhancer
- What do coactivators and corepressors bind to? (2007, 2011, 2012, 2018)
- C region/domain of receptor
- A/B region of receptor
- TAF
- E region of receptor
- 3COOH
- What is the function of corepressors & coactivators? (2018 NIH)
- How do promoters and repressors exert activity? (2011)
- Binding TAFs
- Binding zinc fingers
- Dimerizing hormones
- TAF
- Act on TAFs
- Binding TAFs
What is the binding site of corepressors and coactivators on hormone responsive genes? (2014, 2018 NIH)
- 3’ untranslated region
- CpG
- 5’ upstream region
- Hinge region
- DNA-binding domain
5’ upstream region
What mediates differences in estrogen receptor alpha (ER-alpha) and estrogen receptor beta (ER-beta)? (2013, 2016, 2017, 2018)
Coactivators and corepressors
Presence of TAF-1 on ER-alpha (A/B region)
Where does binding on DNA occur with the hormone response element? (2011)
- On mRNA
- Upstream to 5’ transcription
Upstream to 5’ transcription
What enzyme converts progesterone to dihydroprogesterone? (2017, 2018, 2022)
- 5-alpha-reductase
- 7-alpha-hydroxylase (7-alpha-OH)
5-alpha-reductase
- A mutation in DAX1 most likely results in what? (2003, 2007, 2011, 2012, 2014, 2018, 2018 NIH, 2019, 2021)
- Ovarian failure
- Adrenal hypoplasia/failure
- Delayed puberty
- Failed testicular development
- Failure to express SRY and SOX9
- Sulfatase deficiency
- What happens in males with DAX1 inactivating mutations? (2014, 2016, 2017, 2018)
- What happens in females with DAX1 inactivating mutations? (2014, 2016, 2017, 2018)
- Adrenal hypoplasia/failure
- Adrenal hypoplasia
- Delayed puberty
What enzyme is the rate limiting step of steroid hormone synthesis? (2018 NIH, 2020)
- StAR
- P450scc
- 17-beta-hydroxysteroid dehydrogenase (17-beta-HSD)
- Aromatase
StAR
What is the availability of gonadotropin beta subunits based on? (2020)
- GnRH pulse amplitude
- GnRH pulse frequency
GnRH pulse frequency
What is increasing sialic acid (residue) (silanization) content associated with? (2011, 2017, 2018, 2018 NIH, 2019)
- Diminished half-life
- Diminished clearance/excretion
- Increasing pH
- Decreased bioactivity
Diminished clearance/excretion
What is the biological effect of sialic acid residues? (2013, 2014)
- Increase urinary vs. recombinant hormone
- Increase pH
- Decrease half-life
- Increased during follicular phase
Increased during follicular phase
What setting do you see hyperglycosylated hCG? (2013)
- Pregnancy (first trimester)
- Male
Pregnancy (first trimester)
What setting do you see persistent hyperglycosylated hCG? (2018 NIH)
- Pregnancy
- Mole
- Pituitary small cell carcinoma
- Choriocarcinoma
Both mole and choriocarcinoma
What is the correct order of relative half-lives arranged from greatest to least? (2007, 2009, 2011, 2012, 2014, 2015, 2017, 2018, 2018 NIH)
- hCG>FSH>GnRH>LH
- LH>FSH>hCG>GnRH
- hCG>FSH>LH>GnRH
- FSH>hCG>LH>GnRH
hCG>FSH>LH>GnRH
What hormone receptor class uses phospholipase C (PLC)? (2007, 2011, 2018)
- FSH
- IGF
- TSH
- GnRH
GnRH
What is the least likely reason why hormones readily pass through the cell membrane? (2009, 2018, 2023 NIH)
- High polarity
- Protein binding
- Fat solubility
- Small size
High polarity
What causes failure to iodinate a new protein? (2001, 2007, 2018, 2018 NIH)
- Excess tyrosine residues
- Increased TSH
- No tyrosine residues available
- Abnormal receptor binding
No tyrosine residues available
Where does the TAF act? (2009, 2018, 2018 NIH)
- 5’ untranslated region
- Initiation codon
- 3’ end of DNA
- DNA-binding domain
5’ untranslated region
Which is most true about SHBG? (2018, 2018 NIH)
- Strongly binds progesterone
- Each molecule binds to a single steroid molecule
- Levels increased by IGF-1
Each molecule binds to a single steroid molecule
SHBG is best described by which of the following? (2011)
- Forms a homodimer with binding of a single hormone
- Binds estradiol in greater affinity than testosterone
Forms a homodimer with binding of a single hormone
What increases SHBG? (2003, 2018, 2018 NIH)
- Insulin
- Hyperthyroidism
- Obesity
- Growth hormone
Hyperthyroidism
What decreases SHBG? (2003, 2018, 2018 NIH)
- Estrogen
- T4
- IGF
- Anorexia
IGF
What is necessary for prostaglandin production? (2018, 2018 NIH)
- Lipolysis
- Protein synthesis
- Arachidonic acid
- PGE2
Arachidonic acid
Which has the highest affinity for the corticotropin receptor? (2010, 2011, 2012, 2018, 2018 NIH)
- Androstenedione
- Progesterone
- Mifepristone (RU-486)
- Estrogen
What is the most potent glucocorticoid antagonist? (2016, 2017, 2018, 2018 NIH)
- Mifepristone (RU-486)
- Mifepristone (RU-486)
Which inhibits the progesterone receptor (PR) the most? (2009, 2010, 2018, 2018 NIH)
- Estrogen
- Progesterone
- 17-hydroxyprogesterone (17-OHP)
- Mifepristone (RU-486)
- Danazol
Mifepristone (RU-486)
An orphan receptor (i.e. SF-1, DAX1) is defined by the lack of? (2011, 2018)
- Known gene
- Known ligand
- Known antibodies
- Known DNA-binding site
- Location of gene is unknown
Known ligand
Which of the following is least likely to be an orphan receptor? (2007, 2023 NIH)
- SF-1
- DAX1
- IGF-1
IGF-1
Somatic cell response to androgens is determined by the gene that is present on what chromosome? (2011, 2012, 2014, 2018)
- Xp
- Xq
- Y
- 22
- 13
Xq
What is the production rate of estradiol? (2018)
100-300ug/day
What is the production rate of androstenedione? (2018)
3 mg/day
How do nuclear receptors quickly exert actions? (2011)
- Transcription of mitochondrial DNA
- Duplication of DNA
- Activating transcription of genomic DNA
- Protein synthesis on ribosomes
Activating transcription of genomic DNA
What is the first event after insulin receptor binding? (2003, 2005, 2007, 2018, 2018 NIH)
- (Tyrosine) Auto-phosphorylation of receptor
- Conformational change of cytoplasmic domain of receptor
- Kinase activation of receptor
- Phosphatase activation of receptor
Conformational change of cytoplasmic domain of receptor
A nuclear receptor transmits its signal via? (2023 NIH)
- Translation
- Transcription
- Opening of calcium channels
- Protein kinase A (PKA)
Transcription
What does an insulin receptor defect cause? (2003)
Increased androgen secretion from the ovary
What is the immediate effect after a (steroid) hormone binds DNA? (2014, 2015, 2023 NIH)
- Dimerization
- Ribosomal DNA transcription
- Genomic DNA replication
- mRNA translation
- Genomic (mRNA) transcription
Genomic (mRNA) transcription
What does T3 use as its receptor? (2011, 2012)
- Phospholipase C (PLC)
- cAMP
- Steroid receptor
Steroid receptor
Which of the following has a nuclear receptor? (2009, 2011, 2012, 2018, 2018 NIH, 2022)
- T4
- T3
- FSH
- GnRH
- ACTH
- TRH
T4
T3*** both are correct
Which of the following has a nuclear receptor? (2018 NIH)
- Steroid hormones
- Peptide hormones
Steroid hormones
Which of the following has an intracellular receptor? (2003, 2012)
- T3
- TSH
- FSH
- LH
- hCG
- GnRH
T3
Which of the following has the least number of amino acids? (2007)
- GnRH
- GHRH
- Oxytocin
Oxytocin
The hormone binding domain is important for which of the following? (2007, 2023 NIH)
- HSP binding
- Dimerization
- Nuclear localization
- HSP binding
- Dimerization
*** both are correct
Where is the least likely place to find synthesis of steroids? (2007, 2023 NIH)
- Mitochondrial membrane
- Cytoplasm
- Nucleus
- Endoplasmic reticulum
Nucleus
Which of the following pairs do not act at the same type of receptor? (1999)
- Growth hormone and prolactin
- Insulin and IGF-1
- Progesterone and levonorgestrel
- Inhibin and TGF-alpha
Inhibin and TGF-alpha
Which of the following has a tyrosine kinase receptor? (1999)
- Growth hormone
- ACTH
- FSH
- GnRH
- IGF-1
IGF-1
Where is the steroid receptor DNA-binding domain located? (2012)
5’ upstream region
What initiates the DNA-binding of the hormone receptor? (2001)
Enhancer region
Progesterone action on stromal cell
Where does the most androgen production take place in women? (2010)
- Ovary
- Adrenal
Ovary
What is one of the first events after ligand binding to a tyrosine kinase receptor? (2007)
- G-protein activation
- Phosphorylation
- IP3
Phosphorylation
How are estrogen receptor alpha (ER-alpha) and estrogen receptor beta (ER-beta) different? (2013, 2015, 2017, 2018)
What is the difference in function of estrogen receptor alpha (ER-alpha) and estrogen receptor beta (ER-beta)? (2023 NIH)
- Methylation
- Dimerization
- Co-Binding
- Coregulator binding site
Co-regulators
Co-Binding
Coregulator binding site
The gene that binds to the steroid hormone receptor to cause transcription of the hormone-responsive gene is modulated by transcription factors that act where? (2003)
5’ upstream region
How is estrogen receptor beta (ER-beta) different from estrogen receptor alpha (ER-alpha)? (2019)
Estrogen receptor beta (ER-beta) lacks TAF-1
How are estrogen receptor alpha (ER-alpha) and estrogen receptor beta (ER-beta) the same (most homologous)? (2012, 2013, 2014, 2015, 2018)
- Regulatory
- DNA-binding domain
- Hinge
- Hormone binding domain
- F region
Which domain is most highly conserved in estrogen receptors? (2023 NIH)
- DNA-binding domain
- Hinge
- Ligand binding domain
Which part of the estrogen receptor alpha (ER-alpha) and estrogen receptor beta (ER-beta) are most homologous? (2018)
- A/B
- C
- D
- E
- F
DNA-binding domain
DNA-binding domain
C
Where are zinc fingers located? (2001, 2003, 2023 NIH)
- DNA-binding domain
- Hormone response element
- Ligand binding domain
DNA-binding domain
What type of receptor do growth hormone and prolactin bind to? (2003)
- Tyrosine kinase associated receptor
- JAK/STAT (aka cytokine receptor)
JAK/STAT (aka cytokine receptor)
Which of the following tissues has the highest ratio of estrogen receptor alpha (ER-alpha) to estrogen receptor beta (ER-beta)? (2001, 2003, 2005, 2023 NIH)
- Bone
- Brain
- Uterus
- Ovary
Uterus
Which androgen is most produced by peripheral conversion? (2003, 2005, 2023 NIH)
- Testosterone
- DHEAS
- Androstenedione
- DHEA
Testosterone
ACTH acts by binding to which type of receptor? (2007)
- G-protein, c-AMP
- G-protein, IP3
- Cytokine
G-protein, c-AMP
What does estrogen and progesterone priming lead to in progesterone receptor A (PR-A) and progesterone receptor B (PR-B) knockout mice? (2003, 2010, 2018, 2023 NIH)
- Lordosis
- Endometrial glandular hyperplasia
- Breast development
Endometrial glandular hyperplasia
What does not happen in a mouse model with a progesterone receptor (PR) knockout? (2018)
Decidualization
What is the mechanism of action of progesterone receptor B (PR-B)? (2005, 2007)
- Acts on progesterone elements
- Modulation of estrogen receptor (ER)
- Inhibition progesterone receptor A (PR-A) function by binding to progesterone receptor A (PR-A)
- Inhibiting homodimer formation
- Stimulate progestin-related actions
- Other
Stimulate progestin-related actions
What works though a G protein using phospholipase C (PLC)? (2007)
- GnRH
- Dopamine
- CRH
GnRH
Which is a decapeptide? (2023 NIH)
GnRH
What does CYP2R1 do in the liver? (2023 NIH)
Vitamin D conversion to 25-hydroxyvitamin D
What gene is associated with ovarian development (differentiation)? (2014, 2017, 2018, 2020)
- FOXL2
- HOXA 8/10 (HOX)
FOXL2
Which of the following genes are not involved in promoting ovarian development? (2018)
- WNT4
- RSPO1
- DAX1
- WT1
- FOXL2
WT1
What happens in oocyte meiotic arrest? (2013)
- Inhibiting factor from granulosa cells/functioning granulosa cells
- Gap junctions
Inhibiting factor from granulosa cells/functioning granulosa cells
What happens during embryo development? (2018)
- Demethylation of imprinted genes
- Demethylation of non-imprinted genes
Demethylation of non-imprinted genes
What enzyme is the granulosa cell missing? (2009, 2018, 2023 NIH)
- 3-beta-hydroxylase (3-beta-OH)
- 17-alpha-hydroxylase (17-alpha-OH)
- 17-beta-hydroxysteroid dehydrogenase (17-beta-HSD)
- Aromatase
17-alpha-hydroxylase (17-alpha-OH)
What other malignancy do you check for in a patient with uterine and cutaneous fibroids? (2014, 2015, 2017, 2018, 2019, 2021)
- Renal cell carcinoma
- Colon cancer
- Endometrial cancer
- Medullary thyroid cancer
- Ovarian cancer
Renal cell carcinoma
(HLRCC: Hereditary leiomyomatosis and renal cell cancer)
Patient has a 0.5cm fundal indentation and a 2cm portion of tissue extending from the interostial line into the cavity. What is the diagnosis? (2021)
- Septum
- Bicornuate
- Arcuate
- Didelphys
Septum
What is the least helpful in distinguishing a uterine septum from a bicornuate uterus? (2003, 2005, 2009, 2010, 011, 2012, 2018, 2023 NIH)
- Transvaginal ultrasound (US)
- Sonohysterogram
- HSG
- MRI
- Laparoscopy and hysteroscopy
HSG
What is the most helpful in distinguishing a uterine septum from a bicornuate uterus? (2023 NIH)
- Transvaginal ultrasound (US)
- Sonohysterogram
- HSG
- MRI
- Laparoscopy and hysteroscopy
MRI
What is the most common complication of a unicornuate uterus? (2001, 2018, 2023 NIH)
- First trimester pregnancy loss
- Second trimester pregnancy loss
- Preterm delivery
- Preeclampsia
First trimester pregnancy loss
A patient is found to have a longitudinal vaginal septum. What is the best next step? (2020)
MRI
Hysteroscopy
Laparoscopy
Ultrasound (US)
Ultrasound (US)
MRI
What is least likely to occur with progesterone resistance? (2014, 2016)
- Endometrial synechiae
- Endometrial hyperplasia
- Endometrial polyps
What is least likely to result from inadequate response to progesterone? (2023 NIH)
- Synechiae
- Polyps
- Endometritis
- Luteal defect
Which is the least likely to exhibit progesterone resistance? (2013, 2014)
- Endometriosis
- Endometritis
- Endometrial synechiae
Endometrial synechiae
Synechiae
Endometritis
Endometrial synechiae
What is true about leiomyosarcoma? (2015, 2023 NIH)
- > 10 mitoses per 10 high power fields
- Med 12 somatic mutation
- Some gene mutation that I had never heard of
> 10 mitoses per 10 high power fields
What is true about ethnic differences in fibroids? (2015)
- Black women have younger age of onset than white women
- More aneuploidy in fibroid cells
Black women have younger age of onset than white women
What do you give for fibroids in patients who have bleeding despite OCPs? (2015)
- Levonorgestrel IUD (Mirena)
- High dose progestin
- Uterine artery embolization (UAE)
- Rollerball ablation
Levonorgestrel IUD (Mirena)
What happens at menses? (2015, 2016)
- Vasoconstriction and coagulation
- Stromal breakdown
Stromal breakdown
(Lateral) Fusion defects are due to fusion abnormalities of which of the following? (2011, 2012, 2018, 2023 NIH)
- Mesonephric ducts
- Paramesonephric ducts
- Metanephric ducts
- Wolffian ducts
- Vaginal platea
- Metanephric ducts
- Urogenital sinus
Paramesonephric ducts
For which of the following conditions is an endometrial biopsy least useful for? (2007, 2012, 2023 NIH)
- Chronic endometritis
- RPL workup
- Test for ovulation
- PCOS
- Thickened endometrium >12mm
- Luteal phase deficiency
- Infertility workup
RPL workup
Luteal phase deficiency
Infertility workup
What is the most likely cause of bilateral (proximal) tubal occlusion on HSG? (2011)
- PID
- Tuberculosis
- Tubal spasm
- Salpingitis isthmica nodosa (SIN)
Tubal spasm
Salpingitis isthmica nodosa (SIN) causes which of the following? (2011)
- Hyperplasia of the mucosa
- Inflammation of the tubal epithelium
- Tubal scarring
Tubal scarring
Which of the following is the least useful test for salpingitis? (2010)
- HSG
- Mycoplasma
Mycoplasma
What is the mechanism of growth of fibroids? (2001, 2007, 2023 NIH)
- Acquired genetic abnormality
- TGF-beta dysregulation
- Others
TGF-beta dysregulation
What has uterus didelphys been associated with? (2022)
- Increased spontaneous miscarriage, increased preterm birth
- Increased infertility, increase in preterm birth
- Failure to progress, increase in preterm birth
- Decreased spontaneous abortion, increased preterm birth
Increased spontaneous miscarriage, increased preterm birth
Which of the following gene mutations is associated with mullerian anomalies in genetic females? (2022)
- HOXA13
- WNT4
- LIM1
- MIS
MIS
What is a consequence of an uncorrected vaginal septum in a sexually-active woman discovered incidentally on routine pap testing? (2022)
- Spontaneous miscarriage
- Infertility
- Endometriosis
- Labor dystocia
Labor dystocia
What is the most common finding in conjunction with Mullerian agenesis? (2003)
- Vertebral column abnormalities
- Foot deformity
- Pulmonary stenosis
Vertebral column abnormalities
Diethylstilbestrol (DES) exposure is least likely to cause which of the following? (2003, 2010)
- Cervical dysplasia
- Uterine septum
- Ectopic pregnancy
- Others
Uterine septum
What of the following has the highest likelihood of repair for the fallopian tubes? (2007)
- Extensive ovarian adhesions
- Hydrosalpinx involving 3cm of tube
- Salpingitis isthmica nodosa (SIN)
- Rugation of tubes on HSG
Extensive ovarian adhesions
What type of fallopian tube abnormality is easiest to correct? (2005)
- Bilateral hydrosalpinges
- Salpingitis isthmica nodosa (SIN)
- Tubal rugae on HSG
Bilateral hydrosalpinges
An infertility patient has an 8cm fibroid that is partially submucosal and Hct 22%. What is the best plan of management? (2005)
- Expectant management
- Hysteroscopic myomectomy
- GnRH agonist then abdominal myomectomy
- Others
GnRH agonist then abdominal myomectomy
Which of the following has the worst prognosis? (2023 NIH)
- Uterine septum
- Adenomyosis
- Unicornuate uterus
Uterine septum
Which receptor uses protein kinase C (PKC) as a second messenger? (2022, 2023 NIH)
- CRH
- GnRH
- Activin
GnRH
What activates phospholipase C (PLC)? (2011)
- Dopamine
- GnRH
- CRH
- Insulin
GnRH
What is the first action after GnRH binding? (2018)
- Autophosphorylation of the receptor
- Conformational change in the receptor
- GTP replaces GDP
- Activation of the adenylate cyclase
GTP replaces GDP
Which occurs first when GnRH binds to its receptor? (2011)
- Phospholipase C (PLC) activation
- G protein binding
- Others
G protein binding
What is the first action after GnRH binds its receptor? (2014, 2023 NIH)
- Protein kinase C (PKC)
- cAMP release
- Phospholipase C (PLC)
- IP3 and calcium release
What is initial action of GnRH binding to receptor (2015, 2018)
- Phospholipase C (PLC)
- Protein kinase C (PKC)
- Gq/G11
What is initial action of GnRH binding to receptor (2017)
- IP3/Ca
- Protein kinase C (PKC)
- Gq/G11
Phospholipase C (PLC)
Phospholipase C (PLC)
IP3/Ca vs Gq/G11
What is the intracellular immediate second messenger for GnRH? (2015, 2019)
- Phospholipase C (PLC)
- IP3
- DAG
- Protein kinase C (PKC)
- Protein kinase A (PKA)
Phospholipase C (PLC)
What mediates rapid response in GnRH receptor signaling? (2012)
- Protein kinase C (PKC)
- Protein kinase A (PKA)
- Calcium release
- DAG inhibition
Calcium release
How is GnRH able to exert immediate effects? (2011)
- cAMP
- Nuclear binding
- Calcium
- IP3
- Tyrosine kinase
Calcium
What is the immediate next step after GnRH binds its receptor? (2023 NIH)
Activation of Gq aka G11
What is most likely seen with LH pulses? (2009, 2010, 2011, 2012, 2018, 2023 NIH)
- Highest pulse frequency in the early follicular phase
- Lowest pulse frequency in the mid luteal phase
- Highest pulse amplitude in the early luteal phase
- Lowest pulse amplitude in the late luteal phase
Highest pulse amplitude in the early luteal phase
(Highest pulse frequency in late follicular phase)
What is the first action after FSH and LH bind to their receptor? (2003)
- Autophosphorylation of the receptor
- Conformational change in the receptor
- GTP replaces GDP
- Activation of the adenylate cyclase
Conformational change in the receptor
Speroff Ch 1 pg 39
Hormone-receptor interaction and binding change the a-subunit conformation
GTP replaces GDP on the a-subunit, freeing b- and gamma-subunits
Allows the GTP a-subunit to bind to the catalytic unit of adenylate cyclase, forming the active enzyme.
Which pattern of GnRH secretion is most likely to cause elevation in FSH? (2014, 2023 NIH)
- Low amplitude, low frequency
- Low amplitude, high frequency
- High amplitude, high frequency
- High amplitude, low frequency
High amplitude, low frequency
What is the pattern of GnRH frequency seen in the luteal phase? (2005, 2015, 2023 NIH)
- Low amplitude, low frequency
- Low amplitude, high frequency
- High amplitude, high frequency
- High amplitude, low frequency
High amplitude, low frequency
How does LH pulsatile secretion in the luteal phase compare to the follicular phase? (2003, 2005, 2007, 2009, 2010, 2011, 2012, 2018, 2023 NIH)
- Increased pulse frequency and increased amplitude
- Decreased pulse frequency and decreased amplitude
- Increased pulse frequency and decreased amplitude
- Decreased pulse frequency and increased amplitude
Decreased pulse frequency and increased amplitude
What controls FSH secretion? (2012, 2018, 2023 NIH)
- GnRH pulse amplitude
- GnRH pulse frequency
- Estradiol pulses
- GnRH-LH interaction
GnRH pulse frequency
How does the frequency of LH in the early follicular compare to the late follicular and early luteal? (2017)
Increase in late follicular, decrease in late luteal
What do you measure in a patient with a pituitary adenoma and large feet and hands? (2018)
- ACTH
- FSH
- IGF-1
- TSH
IGF-1
A woman presents with galactorrhea and is found to have a 2cm mass and prolactin 87ng/mL. What is most likely increased? (2023 NIH)
- Growth hormone
- IGF-1
- TSH
- DHEAS
GH
A patient presents with a 10mm adenoma and is found to have hyperprolactinemia. What test will you order next? (2009)
- IGF-1
- TSH
- Cortisol
- FSH
- Alpha subunit
A woman presents with a 2 cm adenoma and prolactin 40 ng/mL. What is an appropriate lab test to order? (2007)
- Growth hormone
- IGF-1
- Big prolactin
- Something with cortisol
IGF-1
Speroff: ~10% of adenomas that secrete prolactin also secrete GH, leading some to recommend measuring the serum IGF-1 concentration, even in women with microadenomas.
Women with pituitary macro-adenomas require additional evaluation (to hCG, TSH, prolactin, FSH, estradiol), including a serum FT4, IGF-1, and AM cortisol level.
What happens with hyperprolactinemia and hypothyroidism? (2016)
- Block GnRH release
- Modify sensitivity
Block GnRH release
What does hyperprolactinemia increase? (2003, 2011)
- Estrogen
- Testosterone
- DHEAS
- DHEA
DHEAS
A pregnant patient has a 7mm mass, bilateral peripheral vision loss, elevated prolactin, and severe headache. How do you manage these findings? (2007, 2011, 2015, 2016, 2017, 2018)
- Bromocriptine
- C-section and transsphenoidal surgery
- Transsphenoidal surgery now
Bromocriptine
A 32wk pregnant woman presents with a 7mm prolactinoma and visual disturbances. Steroids (dexamethasone) were given for fetal lung maturity. What is the next step? (2009, 2011, 2018, 2023 NIH)
- C-section and transphenoidal resection
- Transphenoidal resection
- Bromocriptine
- C-section and bromocriptine
Bromocriptine
A 28yoF who is not trying to conceive presents with amenorrhea, prolactin 45ng/mL, and no galactorrhea. What is the most appropriate treatment? (2007, 2011)
- Bromocriptine
- OCPs
- No treatment
- HRT
OCPs
What is the mechanism of action of metoclopramide? (1999)
- Aromatase inhibitor
- 18-hydroxylase (18-OH) (aldosterone synthase) inhibitor
- 17-hydroxylase (17-OH) inhibitor
- Dopamine antagonist
Dopamine antagonist
What medication does not increase prolactin? (2011)
- Valium
- Levothyroxine (Synthroid)
- Metoclopramide (Reglan)
- Methyldopa
What is least likely to be associated with hyperprolactinemia? (2011, 2023 NIH)
- Diazepam
- Chlorpromazine
- Estradiol
- Levothyroxine (Synthroid)
Levothyroxine (Synthroid)
What is the most potent glucocorticoid? (2013)
- Cortisone
- Progesterone
- Dexamethasone
- Prednisone
Dexamethasone
A woman presents with a pituitary adenoma. She has a normal TSH and prolactin 35ng/mL (normal 5-30ng/mL). What is the most likely diagnosis? (2014, 2015)
- Non-functioning pituitary adenoma
- Craniopharyngioma
Non-functioning pituitary adenoma
What happens if a large pituitary tumor that extends to the left to the cavernous sinus is resected? (2015)
- Hypopituitarism
- CSF rhinorrhea
- Cavernous sinus thrombosis
- Hyperprolactinemia
Hypopituitarism
Ablation of which nucleus causes hypernatremia? (2009, 2011, 2012, 2018, 2023 NIH)
- Optic nucleus
- Arcuate nucleus
- Supraoptic nucleus
- Paraventricular nucleus
- Preoptic nucleus
- Ventromedial nucleus
- Mediolateral nuclues
Supraoptic nucleus
GnRH secretion is most impacted with ablation of which nucleus? (2014)
- Preoptic nucleus
- Paraventricular nucleus
- Ventromedial nucleus
- Arcuate nucleus
Arcuate nucleus
What is least likely to be required to be replaced in panhypopituitarism? (2009, 2011, 2014, 2018, 2023 NIH)
- TSH
- Growth hormone
- Glucocorticoids
- Gonadotropins
- Estradiol
- Mineralocorticoids
Mineralocorticoids
A woman adopts a child and wants to breastfeed. Which of the following, along with nipple stimulation, is most likely to induce lactation? (2014, 2015)
Metoclopramide (Reglan)
What is pituitary feedback conducted via? (2015)
- Portal circulation
- Systemic circulation
Systemic circulation
A woman presents with a pituitary mass and is found to have elevated FSH 42mIU/mL, normal LH, and increased alpha subunits. What is the diagnosis? (2007, 2011, 2012)
- Prolactinoma
- Thyrotroph adenoma
- Gonadotroph adenoma/Non-functioning adenoma
- FSH-oma
- POI
- Pregnancy
Gonadotroph adenoma/Non-functioning adenoma
What is an increased FSH:LH ratio most likely due to? (2005, 2011)
- Increased GnRH pulse frequency
- Decreased GnRH pulse frequency
- Increased GnRH pulse amplitude
Decreased GnRH pulse frequency
A patient presents with a 12 mm adenoma, galactorrhea, normal cycles, and prolactin 35ng/mL. What is the most likely cause? (2007)
- Non-functioning adenoma
- Empty sella syndrome
- Craniopharyngioma
Non-functioning adenoma
A woman’s pituitary stalk is transected. What hormone will increase because of this? (2003, 2007)
Ablation of the median eminence (pituitary stalk) will create a rise in which hormone? (2007, 2023 NIH)
- TSH
- FSH
- Prolactin
- Oxytocin
Prolactin
The ultrashort feedback loop works because of which of the following? (2007, 2023 NIH)
- Retrograde flow
- Capillary fenestrations
- CSF fluid in 3rd ventricle
What is the explanation for the rapid feedback mechanism for pituitary hormones? (2023 NIH)
- Retrograde flow in the portal system
- Brain barrier
- Axo-neuronal transport of hormones
Retrograde flow
Retrograde flow in the portal system
How do pituitary hormones feedback on the hypothalamus? (2023 NIH)
- Systemic circulation
- Pituitary portal vessels
- Tanycytes
Pituitary portal vessels
Which of the following is a typical finding of craniopharyngioma on imaging? (1999)
- Intracranial calcifications
- Suprasellar extension
- Unilateral lesion
- Sellar erosion
Intracranial calcifications
Where is vasopressin made? (2007, 2023 NIH)
Supraoptic and paraventricular nuclei in hypothalamus
Which of the following is least likely to increase prolactin secretion? (2003)
- Sleep
- Exercise
- Suckling
- Others
Others
Ablation of which nucleus leads to changes in ADH? (2011)
- Preoptic nucleus
- Paraventricular nucleus
- Ventromedial nucleus
Paraventricular nucleus
Prolactin secretion is increased by which of the following? (2003)
- Estrogen
- Progesterone
- T4
- Others
Estrogen
IGF-1 works through which type of receptor? (2003)
Tyrosine kinase
Which of the following does not cause elevated prolactin? (2005)
- Hyperthyroidism
- Chronic renal failure
- Cushing disease
- Hypothyroidism
Hyperthyroidism
A 22yo F has an elevated prolactin level and an 8mm pituitary adenoma noted on MRI. What additional test should you order? (2007)
- Growth hormone
- IGF-1
- Cortisol
- TSH
Growth hormone
What causes histamine release? (1999, 2003)
- GnRH antagonists
- GnRH agonists
- Cyproterone acetate
- Spironolactone
- Insulin
GnRH antagonists
What is the most likely to cause hypernatremia if ablated? (2007)
- Amygdala
- Oxytocin cell
- Periventricular nucleus
Periventricular nucleus
What is the most likely to cause a decrease in gonadotropins if ablated? (2007)
- Amygdala
- Arcuate nucleus
Arcuate nucleus
Dopamine inhibits which of the following in vivo? (2003)
- GnRH
- FSH
- LH
- TSH
GnRH
What causes the transition from preantral/secondary to antral follicle? (What is required for antral progression (antrum progression)?) (2001, 2003, 2007, 2009, 2011, 2012, 2013, 2014, 2015, 2018, 2022, 2023 NIH)
- FSH
- GDF-9
- c-KIT
- Kit ligand
- Inhibin
- BMP15
- AMH
- LH
FSH
What directs development from the primary to preantral/secondary follicle? (2005, 2007, 2009, 2018, 2023 NIH)
- GDF-9
- FSH
- Inhibin
- AMH
- LH
GDF-9
What is the role of the granulosa cells in the primordial follicle? (2009, 2011, 2012, 2014, 2015, 2016, 2017, 2018, 2021, 2023 NIH)
- Secrete GDF-9
- Prevent atresia of the oocyte
- Secrete inhibin
- Secrete activin
- Secrete AMH
- To make kit ligand
Prevent atresia of the oocyte
In a primordial follicle, the oocyte is surrounded by which of the following? (2003)
- Single layer of granulosa cells
- Multiple layers of granulosa cells
- Others
Single layer of granulosa cells
What is connexin 37 used for communication between? (2020)
- Granulosa and oocyte
- Theca interna and externa
- Theca cell to granulosa cell
Granulosa and oocyte
When is peak estradiol in the menstrual cycle? (When is the ovary making the most estradiol?) (2019, 2023 NIH)
- Early follicular
- Late follicular
- Early luteal
- Midluteal
- Late luteal
- Menses
Late follicular
When is the maximum expression of the progesterone receptor (PR) during the menstrual cycle? (2023 NIH)
- Menses
- Late follicular
- Late luteal
Late follicular
What is a progesterone >5ng/mL most associated with? (2014)
- Ovulation
- Receptive endometrium
- Ectopic
Ovulation
What progesterone level is most consistent with ovulation? (2015)
- 1.5ng/mL
- 3 ng/mL
- 5 ng/mL
- 10 ng/mL
10 ng/mL
What phase of the menstrual cycle has the highest concentration (maximum expression) of progesterone receptors (PR)? (2001, 2011, 2012, 2013, 2014, 2015, 2016, 2018)
- Early follicular/proliferative
- Late follicular/proliferative
- Early luteal/secretory
- Mid luteal/secretory
- Late luteal/secretory
Late follicular/proliferative
What hormone decreases during luteolysis (regression of the corpus luteum)? (1999, 2003, 2005, 2007, 2009, 2011, 2012, 2013, 2014, 2018, 2023 NIH)
- FSH
- Inhibin A
- Inhibin B
- Activin
- PGF2-alpha
Inhibin A
What peaks in the luteal phase? (2015, 2016, 2018, 2023 NIH)
- Inhibin A
- Inhibin B
Inhibin A
What is least important for oocyte (follicular) development? (2011, 2013, 2014, 2023 NIH)
- BMP15
- GDF-9
- c-KIT
- Follistatin
Follistatin
What is the action of P4 on the endometrium at the start of menses? (2014, 2015)
Withdrawal leads to endometrial breakdown
What is the first event after IGF-1 binds to its receptor? (2007)
Tyrosine kinase phosphorylation
What is the most reliable marker of ovulation? (2016)
- Basal body temperature
- Urinary LH surge
- Progesterone > 3ng/mL 1wk before menses
Progesterone > 3ng/mL 1wk before menses
When does the zona pellucida first form? (2009, 2018, 2023 NIH)
- Primordial follicle
- Primary follicle
- Preantral follicle
- Antral follicle
Preantral follicle
What agent is responsible for theca interna development? (2007, 2018, 2023 NIH)
- GDF-9
- Inhibin
- Activin
- FOXL2
- FSH
- IGF-1
GDF-9
Which enzyme is lowest in theca cells? (2007, 2023 NIH)
- Aromatase
- All other enzymes in steroidogenesis
Aromatase
What enzyme is most likely involved with ovulation/follicular rupture? (2003, 2005, 2007, 2009, 2011, 2018, 2023 NIH)
- PIP2
- BMP15
- Collagenase
- Decrease in PGE2
- Decrease in PGF2-alpha
- Proteinase
- Cumulus hyaluronidase
Which of the following is most likely to cause of follicular rupture? (2007, 2023 NIH)
- Increase in proteoglycan
- Increase in collagenase
- Increase in follicular pressure
Collagenase
Increase in collagenase
What events occur at ovulation? (2018)
- Decrease in plasminogen activator
- Increase in plasminogen activator inhibitor
- Increase in plasminogen activator
- Decrease in matrix metalloproteinases
Increase in plasminogen activator
Granulosa cells in the follicular phase use which enzyme the least? (2007, 2023 NIH)
- Aromatase
- 17-hydroxylase (17-OH)
- P450scc
17-hydroxylase (17-OH)
At what stage does the follicle develop a blood supply? (2011)
- Primordial follicle
- Preantral follicle
- Antral follicle
- Atretic follicle
Preantral follicle
What happens to the follicle immediately prior to ovulation? (2014, 2015, 2016)
- Oocyte maturation inhibitor expressed
- Resumption of meiosis II
- Increase in prostaglandin
Increase in prostaglandin
At what stage does vascularization of the follicle start? (2009)
- Primary follicle
- Secondary follicle
- Antral follicle
Secondary follicle
What precursor must you have when going from cholesterol to estradiol? (2023 NIH)
- 17-hydroxyprogesterone (17-OHP)
- Testosterone
- Pregnenolone
- Progesterone
Testosterone
Which of the following is most likely in regard to the menstrual cycle? (2023 NIH)
- Progesterone in mid-cycle augments estradiol; feedback on LH
- Estradiol in early follicular phase increases GnRH receptors at pituitary
Progesterone in mid-cycle augments estradiol; feedback on LH
Where is the spindle most likely to be after extrusion of the 2nd polar body? (2021)
- Near 2nd polar body
- Opposite 2nd polar body
- Degraded
- Outside oocyte
- Near 2nd polar body
When does the cortical reaction occur? (2018)
- Sperm bind ZP2
- Sperm binds ZP3
- Sperm fusion to oocyte membrane
Sperm fusion to oocyte membrane
During implantation and uterine invasion, what cell gets replaced in trophoblast invasion? (2013, 2014, 2015, 2017, 2018)
- Vascular endothelium
- Endometrium
- Myometrium
Vascular endothelium
In pregnancy, trophoblast replaces which of the following? (2005, 2014, 2015)
- Glandular epithelium
- Luminal epithelium
- Stroma
- Endometrial glands
Luminal epithelium
What does the cytotrophoblast/trophoblast replace? (2007, 2009)
- Stroma
- Endothelium
- Decidua
- Myometrium
Endothelium
What is a progesterone receptor-regulated (PR-regulated) gene essential for implantation and decidualization? (2018)
HOXA8, HOXA10
How many chromosomes in an MI oocyte? (2009)
- 46 double chromatids
- 46 double chromosomes
How many chromosomes in an MI oocyte? (2009)
46 double chromosomes
How many chromosomes are there in prophase of meiosis I? (2003)
- 23 single-stranded chromosomes
- 23 double-stranded chromosomes
- 46 single-stranded chromosomes
- 46 double-stranded chromosomes
- 92 single-stranded chromosomes
46 double-stranded chromosomes
How many chromosomes are in an MI oocyte? (2018, 2023 NIH)
- 23
- 46
- 92
- Depends on which stage in meiosis I
46
How many chromatids are in an MII oocyte? (2007, 2010, 2023 NIH)
23
46
92
46
What is the number of chromatids present in a metaphase II oocyte? (2007)
- 23 single
- 46 single
- 23 double
- 46 double
46 single
What is the chromosome number of a euploid MII oocyte? (2007, 2023 NIH)
22
23
45
46
23
What is the number of chromosomes when an oocyte is arrested in meiosis I? (2009)
23 double stranded
46 double stranded
46 double stranded
What is the number of chromosomes in the primary oocyte? (2001)
- 23
- 46
- 92
46
What is the number of chromatids present in the oocyte immediately after ovulation? (2005, 2023 NIH)
- 23
- 46
46
What is the number of chromosomes present in the oocyte just prior to ovulation? (2005, 2023 NIH)
- 23 double
- 46 double
46 double
When does the oocyte complete meiosis II? (2018)
- Ovulation
- LH surge
- Fertilization
- Acrosome reaction
- Cortical granule reaction
Fertilization
What least characterizes the inner cell mass (ICM)? (2010, 2011, 2012, 2018)
- Proliferation
- Pluripotency
- Apoptosis
- Exponential growth
- hCG production
hCG production
Mesenchymal stem cells are? (2011, 2012, 2018, 2023 NIH)
- Totipotent
- Pluripotent
- Multipotent
- Unipotent
Multipotent
What sugar is used when the embryonic genome turns on (is activated)? (What is the preferred nutrient when the maternal genome is not longer governing the development of the oocyte) (2007, 2010, 2012)
- Glucose
- Lactate
- Pyruvate
Glucose
Precompaction (Day 1-3) prior to embryonic genome activation, primary energy source is anaerobic with lactate and pyruvate
Post compaction (Day 4-6) after embryonic genome activation, embryo switches to aerobic with increased glucose utilization
What is the first step in sex determination? (What is the first embryonic event leading to the development of a male phenotype?) (Which is the earliest requirement for normal male gonadal development?) (1999, 2003, 2005, 2010, 2012)
- Y chromosome
- SRY (TDF) gene expression/activation
- Testosterone production
- AMH
SRY (TDF) gene expression/activation
Which hormone most likely keeps immune suppression down for implantation? (2007, 2023 NIH)
- ACTH
- Cortisol
- Progesterone
- Estrogen
Progesterone
The oocyte is in which of the following stages just prior to fertilization? (2003, 2005, 2007, 2023 NIH)
- Prophase I
- Prophase II
- Metaphase I
- Metaphase II
Metaphase II
What is the first thing that happens in human embryos at implantation? (2023 NIH)
- Apposition
- Invasion
- Adhesion
Apposition
Which least accurately characterizes maternal thyroid physiology? (2003)
- Increased TSH secretion
- Decreased T4 secretion
Decreased T4 secretion
What enzyme does the fetus lack? (2013, 2015, 2017, 2019, 2022)
- 3-beta-hydroxysteroid dehydrogenase (3-beta-HSD)
- 17-alpha-hydroxylase (17-alpha-OH)
3-beta-hydroxysteroid dehydrogenase (3-beta-HSD)
What enzyme does the placenta lack?
17-alpha-hydroxylase (17-alpha-OH)
Rank corticosteroids in order of increasing potency
“Cold Hands Palpate My Dead Body”
Cortisone
Hydrocortisone
Prednisone
Methylprednisone
Dexamethasone
Betamethasone
Which hormone decreases mid-gestation/second trimester in a male fetus? (2009, 2010, 2011, 2012, 2013, 2015, 2017, 2018, 2019)
- LH
- TSH
- Prolactin
- Estrogen
- Testosterone
- ACTH
- Prolactin
- Growth hormone
- hPL
- E2
LH
Which hormone is least likely to decrease mid-gestation/second trimester? (2007)
- LH
- ACTH
- CRH
- Prolactin
Prolactin
All of the following will rise in pregnancy except? (1999, 2012, 2013, 2018, 2023 NIH)
- hPL
- CRH
- ACTH
- Estriol
- Progesterone
- Growth hormone
- Prolactin
- Cortisol
- T4
Growth hormone
Where is estetrol made? (2014, 2018)
- Fetal adrenal
- Fetal ovary
- Fetal liver
- Placenta
Fetal liver
Where is proopiomelanocortin (POMC) found? (2014, 2018)
Placenta
Placenta
What makes (increases) maternal (serum) CRH in pregnancy? (2003, 2005, 2007, 2009, 2010, 2011, 2012, 2013, 2014, 2015, 2018, 2023 NIH)
- Maternal hypothalamic activity
- Maternal pituitary activity
- Fetal hypothalamic activity
- Placental function
- Decidua
Placental function
What doesn’t come from POMC? (2015)
- Enkephalin
- Beta lipotropin
- Beta endorphin
- ACTH
- MSH
- Melatonin
Melatonin
What is the differential diagnosis of high estriol? (2018)
- Acute fetal hypoxia
- Multiple gestation
- Risk for preterm labor/pending labor
- CAH
Multiple gestation
CAH
What is the differential diagnosis of low estriol? (2018)
- Impending or present fetal demise
- Adrenal hypofunction
- ACTH deficiency
- Placental sulfatase deficiency
- Placental aromatase deficiency
- Drug-related effects (material corticoid therapy)
- Anencephaly
- Best answer per MFM: anencephaly
- Next best answer: present fetal demise
- 3rd best answer: placental sulfatase deficiency
What allows increased communication between cells at the time of delivery? (2014, 2017)
- Increased gap junctions
- Increased P4
Increased gap junctions
What happens at parturition? (2015)
- Functional depletion of progesterone receptors (PR)
- Increased estradiol in myometrium
- Decreased cortisol
Functional depletion of progesterone receptors (PR)
What triggers parturition? (2015)
- Fall in P4 and E2
- Depletion of progesterone receptor B (PR-B)
Depletion of progesterone receptor B (PR-B)
What causes virilization of a female fetus? (2023 NIH)
Danazol
What is the most common cause of maternal virilization in the first trimester? (2003, 2005, 2010, 2011, 2012, 2014, 2015, 2016, 2018, 2022)
- Sertoli-Leydig cell tumor
- Hyperreactio luteinalis (theca-lutein cysts)
- Luteoma of pregnancy
- Aromatase deficiency
- SHBG decrease
- PCOS
- Exogenous androgens
Luteoma of pregnancy
A pregnant woman has a testosterone (androgen) producing tumor. What is the most likely phenotype for a female fetus? (2005, 2011, 2012, 2015, 2018)
- Normal
- Virilized fetus
- Ambiguous genitalia
- Mixed gonadal dysgenesis
Normal
A woman has a twin pregnancy and develops hirsutism, what is the most likely cause? (2003, 2005, 2018)
- Nonclassical CAH
- Luteoma of pregnancy
- Hyperreactio luteinalis (theca-lutein cysts)
- Aromatase deficiency
Hyperreactio luteinalis (theca-lutein cysts)
A woman with hyperthecosis is least likely to have which of the following? (2009)
- Hypertension
- Dyslipidemia
- Acanthosis nigricans
- Virilization
Hypertension
The placenta synthesizes progesterone from what precursor? (2012, 2018)
- Maternal LDL
- Maternal HDL
- Acetate
- Fetal DHEAS
Maternal LDL
On a molar basis (In a molar pregnancy), what androgen does the theca cell produce the most? (2003, 2005, 2007, 2009, 2011, 2012, 2014, 2018, 2023 NIH)
- Androstenedione
- Testosterone
- DHEAS
- DHEA
Androstenedione
What is the least likely to cross placenta? (2005, 2018)
- T4
- TSH
- TRH
- Propylthiouracil (PTU)
- Thyroid antibody
TSH
What substances do not cross placenta? (2018)
- Glucose
- Amino acids
- Ketones
- Insulin
- IGF-1
Insulin
What is the best test to diagnose Addison disease in pregnancy? (2005, 2012, 2018)
- ACTH stimulation test
- Metyrapone test
- Insulin tolerance test (ITT)
- Glucose tolerance test (GTT)
ACTH stimulation test
What is the least likely mechanism of AIS at the receptor level? (2013, 2018)
Altered bioactivity of testosterone
What does a defect in the androgen receptor result in? (2015)
- AIS
- Klinefelter syndrome
- SMA
- 17-alpha-hydroxylase deficiency (17-alpha-OHD)
AIS
What is the inheritance pattern of AIS? (2013, 2020, 2021, 2022)
- Autosomal dominant
- Autosomal recessive
- X-linked dominant
- X-linked recessive
X-linked recessive
What is the most important requirement (principal determinant) for Wolffian duct differentiation? (2007, 2011, 2012, 2013, 2014, 2018)
- Testosterone
- SRY
- Y chromosome
- Androstenedione
- AMH
- DHT
Testosterone
What are the internal structures of a 46,XY individual with 17-alpha-hydroxylase deficiency (17-alpha-OHD)?
No internal structures (Unless testes is an option then pick that because these individuals will have some kind of cryptorchidism)
What is the pattern of Wolffian and Mullerian structures in complete AIS? (2014, 2015, 2017, 2018)
Normal Wolffian structures
Normal Mullerian structures
Both Wolffian and Mullerian structures
No Wolffian or Mullerian structures
No Wolffian or Mullerian structures
What is the gene loci responsible for androgen resistance? (2011)
- 46,XX
- 46,XY (13/15)
- 46,XY (11)
- 46,XY (t5)
46,XY (11)
Abnormality on which chromosome can cause (clinical features of AIS)? (2013, 2014)
- X
- Y
- 13
X
What chromosome is the androgen receptor gene located? (2014, 2018, 2023 NIH)
- Xq
- Xp
Xq
A 27yoF postpartum presents with fatigue and constipation, yellow coloring, TSH normal 1mIU/mL, T4 0.3mcg/dL. What is the best next step (2007)
- TRH stimulation test
- Prolactin
- Check T3 resin uptake
- MRI
Check T3 resin uptake
What causes most of insulin resistance in pregnancy? (2007)
hPL
What changes in fetal hormones in the third trimester? (2011)
- TSH can’t remember if it was increase or decrease
- Decreased FSH***
- Increased LH
- Cortisol can’t remember if it was increase or decrease
When is the soonest thyroid levels should be checked after an increase in levothyroxine (Synthroid) dose? (2012)
- 1wk
- 2wk
- 4wk
- 8wk
6-8 weeks
What is the earliest time to make a possible change in thyroid medication doses in pregnancy? (2014)
- 1wk
- 2wk
- 4wk
- 8wk
Should initiate as soon as pregnancy diagnosed, but increased need for T4 by 5 weeks
What are the normal changes to maternal thyroid function in the first trimester of pregnancy? (2010, 2012, 2014)
Increase in TSH, increase in T4
Increase in TSH, decrease in T4
Decrease in TSH, decrease in T4
Decrease in TSH, increase in T4
Decrease in TSH, increase in T4
(Due to mimicry of hCG)
What is the most likely cause of increased maternal urinary estriol? (2014)
- Placental sulfatase deficiency
- Fetal P450c17 deficiency
- Fetal 21-hydroxylase deficiency (21-OHD)
- Fetal 11-beta-hydroxylase deficiency (11-beta-OHD)
Fetal 21-hydroxylase deficiency (21-OHD)
What causes high estriol in pregnancy? (2013, 2014, 2015, 2016, 2017, 2018)
- Sulfatase
- Aromatase
- 21-hydroxylase deficiency (21-OHD)
21-hydroxylase deficiency (21-OHD)
What is the immediate precursor to estradiol in pregnancy? (2018)
- Testosterone
- DHEAS
- 16-hydroxy-dehydroepiandrosterone-sulfate (16-OH-DHEAS)
- 16-hydroxy-androstenedione (16-OH-androstenedione)
- Estriol
Testosterone
A pregnant woman at 16wk presents with low estriol levels on screening. What is the best next step? (2021)
- Check sulfated estriol
- Amnio for 17-hydroxyprogesterone (17-OHP) level
- cffDNA
- CVS
- Ultrasound (US)
Ultrasound (US)
What enzyme does the placenta lack? (1999, 2001, 2003, 2005, 2007, 2010, 2011, 2012, 2013, 2015, 2018, 2019, 2020, 2022)
- 17-alpha-hydroxylase (17-alpha-OH) and 17,20-lyase
- 17-beta-hydroxysteroid dehydrogenase (17-beta-HSD)
- 3-beta-hydroxysteroid dehydrogenase (3-beta-HSD)
- Aromatase
- Sulfatase
- 11-beta-hydroxylase (11-beta-OH)
17-alpha-hydroxylase (17-alpha-OH) and 17,20-lyase
Which is a fetal enzyme needed for production of estriol? (2018)
- 15-alpha-hydroxylase (15-alpha-OH)
- 16-alpha-hydroxylase (16-alpha-OH)
- 3-beta-hydroxysteroid dehydrogenase (3-beta-HSD)
- 17-beta-hydroxysteroid dehydrogenase (17-beta-HSD)
- 17-alpha-hydroxylase (17-alpha-OH) and 17,20-lyase
16-alpha-hydroxylase (16-alpha-OH)
What enzyme is exclusively produced by the fetal adrenal? (2018)
- 16-alpha-hydroxy-dehydroepiandrosterone-sulfate (16-alpha-OH-DHEAS)
- 16-alpha-hydroxy-dehydroepiandrosterone (16-alpha-OH-DHEA)
- Estriol
- Others
16-alpha-hydroxy-dehydroepiandrosterone-sulfate (16-alpha-OH-DHEAS)
What does a defect in the androgen receptor result in? (2015)
- AIS
- Klinefelter syndrome
- SMA
- 17-alpha-hydroxylase deficiency (17-alpha-OHD)
AIS
What is the least likely mechanism of AIS at the receptor level? (2013, 2018)
Altered bioactivity of testosterone
What is the likely mechanism of complete AIS? (2018)
- Mutation in androgen receptor gene
- Mutation in 5-alpha-reductase enzyme
- Failure of SF-1 to interact with SRY
- Dysgenesis of gonad
Mutation in androgen receptor gene
Translocation of the SRY from the Y to the X chromosome is due to the fact that the SRY is most closely located near which of the following? (2021, 2022)
- Pseudoautosomal region
- AZFa
- AZFb
- AZFc
Pseudoautosomal region
When should gonads be removed in AIS?
In patients with complete AIS, gonadectomy generally is best delayed until after puberty is completed (approximately age 16–18) because pubertal development generally proceeds more smoothly in response to endogenous hormone production and because the overall risk for tumor development is quite low (5–10%), particularly before puberty.
What is the pseudoautosomal region?
Where is it located?
Why is it significant?
The “pseudoautosomal” region constitutes only approximately 5% of the entire Y chromosome and is the only region that normally pairs and recombines during meiosis.
A single copy of the one gene most critical to testis differentiation, SRY (Sex-determining Region on Y), is located on the distal short arm of the Y (Yp11.3), immediately adjacent to the pseudoautosomal region.
Because it is so close to the SRY gene, translocation of SRY to the X chromosome is more likely
An infant presents with a 46,XX karyotype, (right) labioscrotal mass, and ambiguous genitalia. What is the MOST likely diagnosis? (2005, 2007, 2009, 2011, 2012, 2013, 2015, 2016, 2017, 2018, 2019, 2022)
- 46,XX, ovotesticular DSD (true hermaphroditism)
- 46,XX, testicular DSD (sex reversal; de la Chapelle syndrome)
- 46,XX, aromatase deficiency
- 46,XX, AMH deficiency
- Mixed gonadal dysgenesis
- Complete AIS
46,XX, ovotesticular DSD (true hermaphroditism)
An infant presents with ambiguous genitalia. There is no labioscrotal mass on exam. What is the MOST likely diagnosis if the karyotype is 46,XX? (2013, 2018)
- CAH
- Finasteride exposure during pregnancy
- Luteoma during pregnancy
- 46,XX, testicular DSD (sex reversal; de la Chapelle syndrome)
- 46,XX, ovotesticular DSD (true hermaphroditism)
CAH
An infant presents with ambiguous genitalia and a labioscrotal mass. What is the LEAST likely diagnosis if the karyotype is 46,XX? (2007, 2013, 2014, 2018)
- 5-alpha-reductase deficiency
- 46,XX male
- 46,XX, ovotesticular DSD (true hermaphroditism)
- CAH
CAH
A woman presents with fatigue, increased pigmentation, weight loss, and no androgen excess. What is the diagnosis? (2011, 2012, 2018)
- Autoimmune disease
- Diabetes
- Late onset 21-hydroxylase deficiency (21-OHD)
Autoimmune disease
A young man presents with hyperpigmentation, hyponatremia, and hyperkalemia. What is the most likely etiology? (2014, 2015)
- Autoimmune disease
- 3-beta-hydroxysteroid dehydrogenase deficiency (3-beta-HSDD)
- 11-beta-hydroxysteroid dehydrogenase deficiency (11-beta-HSDD)
- Sjogren disease
Autoimmune disease
What is the most likely phenotype in an activating LH receptor defect (LH receptor activating mutation, constitutive activation of the LH receptor)? (2003, 2005, 2009, 2011, 2012, 2014, 2015, 2016, 2017, 2018, 2020, 2021, 2022)
- Female with PCOS and elevated FSH
- Normal female
- Normal puberty
- Precocious puberty in a male (with low FSH and arrested spermatogenesis)
- Isosexual precocious puberty in female
- Precocious female puberty
Precocious puberty in a male (with low FSH and arrested spermatogenesis)
What is the risk of mortality from aortic dissection in pregnancy and postpartum in patients with Turner syndrome? (2018)
- 2/10
- 2/100
- 2/1000
- 2/10000
2/100 (2%)
What is the gene defect causing short stature in Turner syndrome (46,X, or 46,X/46,XX mosaic, or 46,X/46,XY mosaic)? (2018)
- SHOX
- SRY
- FMR1
- Anosmin-1
- AR (androgen receptor)
SHOX
A patient presents with primary amenorrhea and short stature. What is the most likely diagnosis? (2010)
Turner syndrome
An 18yoF with Tanner stage 5 breast development, pubic hair development, and primary amenorrhea. What is the best first test? (2005, 2020)
- Testosterone
- FSH
- 17-hydroxyprogesterone (17-OHP)
- Karyotype
FSH
What is the most likely phenotype in an AMH receptor defect? (2019)
- Persistence of Mullerian structures
- POI
- Precocious puberty
Persistence of Mullerian structures
An adolescent girl with primary amenorrhea is found to have Mullerian agenesis and a 1cm vaginal dimple. Which is the most appropriate treatment when she is ready to become sexually active? (2003, 2010)
- Frank dilators
- Ingram procedure
- McIndoe procedure
- Vecchietti procedure
Frank dilators
What is least likely with an 11beta-hydroxylase (11-OH) deficiency mutation? (2014)
- Hypertension
- Hyperkalemia
- Hypernatremia
Hyperkalemia
What do you find in male newborn with 21-hydroxylase deficiency (21-OHD)? (2012)
- Scrotal testes
- Hypospadias
- Café-au-lait spots
- Ambiguous genitalia
Scrotal testes
What is the mechanism of complete AIS? (2009, 2014, 2015)
- Hormone cannot bind receptor (decrease biologic activity)/Delta-5 androgens do not bind to receptor
- Receptor cannot bind to DNA?
- Other things about the receptor but nothing about a receptor defect
Hormone cannot bind receptor (decrease biologic activity)/Delta-5 androgens do not bind to receptor
What is an activating receptor mutation most likely to cause? (2012, 2018, 2023 NIH)
- Kallmann syndrome
- AIS
- CAH
- MEN2
- Myotonic dystrophy
MEN2
A 5yo African American girl presents with has pubic hair, Tanner stage 1 breasts, and growth 60th percentile. What do you do next? (2003, 2012, 2021)
- Estradiol level
- DHEA
- Testosterone
- Bone age
Bone age
A 5yoF presents with Tanner stage 3 pubic hair (pubarche). What is the next step? (2005, 2009, 2010, 2011, 2012, 2018)
- Depot medroxyprogesterone acetate (DMPA) (Depo-Provera)
- Depot GnRH
- Check testosterone level
- Check 17-hydroxyprogesterone (17-OHP)
- Pelvic ultrasound (US)
- Pituitary MRI
Check 17-hydroxyprogesterone (17-OHP)
A child presents with isolated precocious adrenarche. What is the best test to order? (2012)
17-hydroxyprogesterone (17-OHP)
17-hydroxyprogesterone (17-OHP)
What level do you need to optimize in the follicular phase of a patient with 21-hydroxylase deficiency (21-OHD)? (2021)
- Progesterone
- 17-hydroxyprogesterone (17-OHP)
- DHEAS
- Testosterone
Progesterone
What is most likely to cause a false positive 17-hydroxyprogesterone (17-OHP)? (2005, 2009, 2011, 2010, 2012, 2018)
- Not fasting/Postprandial
- Recent LH surge/Recent ovulation
- Pulsing of 17-hydroxyprogesterone (17-OHP)
- Luteal phase
Diurnal variation
Luteal phase
A 12yo presents with complete ovarian failure (POI). How do you supplement hormones? (2023 NIH)
- Slow incremental estrogen
- Estrogen and progesterone
- Growth hormone
Slow incremental estrogen
What is the cause of Turner mosaicism? (2020)
- Postzygotic mitotic error
- Oocyte mitotic error
- Maternal meiotic error
- Paternal meiotic error
Postzygotic mitotic error
Where does the error occur in mosaic Turner 46,X/46,XX? (2018, 2022)
- Meiosis II (paternal)
- Meiosis II (maternal)
- Mitosis after fertilization
Mitosis after fertilization
17yo presents with amenorrhea, normal electrolytes, and Tanner stage 3 breasts. Where is the most likely defect? (2007)
5-alpha-reductase
Androgen receptor
Androgen receptor
A young girl is found to have elevated FSH and progesterone. What else do you expect to find on evaluation? (2013)
A 17yo with primary amenorrhea and female external genitalia is found to have FSH 50mIU/mL, and progesterone 25ng/mL. What else do you expect to find on evaluation?
(2007, 2011, 2012, 2018, 2023 NIH)
- Hypercortisolism
- Hypertension
- Absent uterus
Hypertension
What is the most likely way to diagnose 17-alpha-hydroxylase deficiency (17-alpha-OHD) in a child? (2019)
- Hyponatremia
- Hypokalemia
- Premature adrenarche
- Salt wasting
Hypokalemia
What is most likely to be seen in a teenager with 17-alpha-hydroxylase deficiency (17-alpha-OHD)? (2011)
- Hypertension
- Hyperkalemia
- Hirsutism
Hypertension
What is the least likely way to diagnose 17-alpha-hydroxylase deficiency (17-alpha-OHD) in a child? (2005, 2011)
- Hyponatremia
- Hypokalemia
- Hypertension
- Premature adrenarche
- Salt wasting
Hyponatremia/salt wasting
What enzyme deficit will not cause salt wasting? (2014, 2015, 2018)
- 17-alpha-hydroxylase (17-alpha-OH)
- 11-beta-hydroxylase deficiency (11-beta-OHD)
17-alpha-hydroxylase (17-alpha-OH)
What enzyme deficiency is least likely to cause precocious puberty? (2013, 2014, 2023 NIH)
- 17-hydroxylase deficiency (17-OHD)
- 3-beta-hydroxysteroid dehydrogenase deficiency (3-beta-HSDD)
- 17-beta-hydroxysteroid dehydrogenase deficiency (13-beta-HSDD)
- Aromatase deficiency
17-hydroxylase deficiency (17-OHD)
What is the best management of long-term amenorrhea in nonclassical CAH? (2016, 2017, 2018)
- OCPs
- Hydrocortisone
OCPs
What is the order of hormone repletion in thyroid and adrenal disorders? (2007, 2009, 2013, 2015, 2016, 2018)
- Glucocorticoid then mineralocorticoid and last thyroid replacement
- Mineralocorticoids and glucocorticoids; then thyroid
- Thyroid, then mineralocorticoids and glucocorticoids
- Glucocorticoids, then thyroid, then mineralocorticoids
- Only glucocorticoids and mineralocorticoids
What do you first treat in a patient with both adrenal insufficiency and hypothyroidism? (A patient is diagnosed with both Addison disease and Hashimoto thyroiditis. What is the next best step in treatment?) (2003, 2005, 2007, 2011, 2012, 2014, 2023 NIH)
- Replace all three at once
- Thyroid and cortisol then watch see if mineralocorticoid replacement is needed
- Thyroid then watch and see if others require replacement
- Replace glucocorticoids and mineralocorticoids then thyroid
Glucocorticoid
then mineralocorticoid
and last thyroid replacement
How do you monitor mineralocorticoid activity? (2015, 2017)
How do you monitor and titrate someone on mineralocorticoid replacement (for adrenal insufficiency)? (2011, 2013, 2014)
- Plasma renin level
- Sodium
- Potassium
- 17-hydroxyprogesterone (17-OHP)
Plasma renin level
What are patients with 46,XY complete gonadal dysgenesis (Swyer syndrome) most likely to have? (2014)
- Normal breast development
- Vagina
- Low FSH
- Normal sperm
- High P4
Vagina
What is the most likely etiology of a 46,XX male? (2005, 2009, 2011, 2012, 2014, 2015, 2017)
- Recombination or translocation
- Inversion
- Nondisjunction
- Deletion
What type of event causes a 46,XX male? (2018)
- Chromosomal substitution
- Abnormal recombination event during embryonic development
- Abnormal recombination/translocation during male meiosis
- Developmental mutation
Recombination or translocation
Abnormal recombination/translocation during male meiosis
What region on the Y chromosome is most likely affected by crossover? (2013, 2014, 2018)
- Pseudoautosomal region
- SRY
- Yp
- AZF region
Pseudoautosomal region
A patient presents with hirsutism and 17-hydroxyprogesterone (17-OHP) >800. What test would you do next? (2016, 2017)
- Cosyntropin stimulation test
- 24-hour urinary cortisol
- CRH stimulation
Cosyntropin stimulation test
What is the effect of flutamide in pregnancy? (2018)
- Teratogen – abnormalities in male fetus
- Teratogen – decreased estrogens in female fetus
- No effect
- Teratogen – abnormal prostate function later in life
Teratogen – abnormalities in male fetus
What is the first determinant of male development? (2009, 2018)
- Differentiation of Leydig cells
- Sertoli cell development from SF-1 and SRY
- Peritubular myoid cell development
- Testicular formation
Sertoli cell development from SF-1 and SRY
What is the next step when streak gonads are found during an appendectomy? (2018)
- Removal
- Sequence AR gene
- Sequence SRY gene
- Karyotype
Karyotype
What is the most common chromosomal abnormality in gonadal dysgenesis? (2003, 2018)
- 46,XY complete gonadal dysgenesis (Swyer syndrome)
- 46,X, Turner syndrome
- 46,XXY, Klinefelter syndrome
- 46,X, 46,XY Turner mosaic
46,X Turner syndrome
Which has the highest risk for tumor formation? (2018)
- Mixed gonadal dysgenesis
- 46,XY complete gonadal dysgenesis (Swyer syndrome)
- 45,X/46,XY (Turner mosaic)
- AIS
- 45,X
46,XY complete gonadal dysgenesis (Swyer syndrome)
What genetic mutation can cause abnormal female structures in 46,XX? (2012, 2018)
- SRY
- SOX9
- FGF-9
- GDF-15
- FOXL2
FOXL2
How do you clinically define 46,XX DSD? (2018)
- SRY and cell expression
- Absence of Mullerian ducts and presence of Wolffian ducts
- Presence of ovarian tissue (follicles) and testicular tissue (seminiferous tubules)
Presence of ovarian tissue (follicles) and testicular tissue (seminiferous tubules)
Picture of ambiguous genitalia, with a 46,XY karyotype. What is the least likely diagnosis? (2018)
- 46,XY complete gonadal dysgenesis (Swyer syndrome)
- CAH
- AMH deficiency
- AIS
- 5-alpha-reductase deficiency
- Testicular regression syndrome
- 17-hydroxylase deficiency (17-OHD)
- 17-beta-hydroxysteroid dehydrogenase deficiency (17-beta-HSDD)
CAH
Usually normal male at birth
What do you find in male newborn with 21-hydroxylase deficiency (21-OHD)? (2012)
- Scrotal testes
- Hypospadias
- Café-au-lait spots
- Ambiguous genitalia
Scrotal testes
What are you most likely to see with 21-hydroxylase deficiency (21-OHD)? (2012, 2018, 2023 NIH)
- Decreased progesterone and 17-hydroxyprogesterone (17-OHP)
- Decreased deoxycorticosterone (DOC) and 11-deoxycortisol
- Increased aldosterone
- Low cortisol
Decreased deoxycorticosterone (DOC) and 11-deoxycortisol
Low cortisol
What test is best for diagnosing nonclassical 21-hydroxylase deficiency (21-OHD)? (2015)
- ACTH stimulated 17-hydroxyprogesterone (17-OHP)
- ACTH stimulated 17-hydroxypregnenolone
- 17-hydroxypregnenolone
ACTH stimulated 17-hydroxyprogesterone (17-OHP)
What is most important for testosterone production in the male fetus after 20wk? (2014, 2018)
- Maternal androstenedione production
- Fetal LH
Fetal LH
What is least likely to be seen in 11-beta-hydroxylase deficiency (11-beta-OHD)? (1999, 2003, 2011, 2012, 2014, 2015, 2018)
- Hypertension
- Elevated 11-deoxycortisol
- Elevated ACTH
- Salt wasting
- Hypokalemia
- Precocious puberty
- Virilization
- Anovulation
- Irregular cavity with adhesions
Salt wasting
What is the inheritance pattern in 17-alpha-hydroxylase deficiency (17-alpha-OHD)? (2018)
Autosomal recessive (AR)
Why is ACTH elevated (in 17 alpha hydroxlase deficiency)? (2018)
Cortisol is a much stronger glucocorticoid than corticosterone or deoxycorticosterone (DOC)
What class of steroids are produced in excess in 17-alpha-hydroxylase deficiency (17-alpha-OHD)? (2018)
- Mineralocorticoids
- Glucocorticoids
Mineralocorticoids
A 17 yo patient presents with primary amenorrhea, 95th percentile growth, Tanner stage 1 breasts, and Tanner stage 1 pubic hair. What is the enzyme defect ? (2018, 2023 NIH)
17-alpha-hydroxylase deficiency (17-alpha-OHD)
A 16yo presents with Tanner stage 1 hair and breasts, FSH 35 mIU/mL, progesterone 23 ng/mL. What is the most likely etiology? (2011)
- Isolated gonadotropin deficiency
- FSH receptor defect
- Turner syndrome
- 17-hydroxylase deficiency (17-OHD)
17-hydroxylase deficiency (17-OHD)
A 17 yo F presents with Tanner stage 1 everything, 90th percentile height and weight, a small uterus, and no adnexal masses. What is the most likely diagnosis? (2010, 2023 NIH)
- Isolated gonadotropin deficiency
- DAX1 mutation
- AIS
DAX1 mutation
A 16 yo presents with Tanner stage 1 everything, 90th percentile height, a uterus, and a cervix. What is the most likely diagnosis? (2023 NIH)
46,XY complete gonadal dysgenesis (Swyer’s)
A 16yo presents with primary amenorrhea, 90th percentile height, Tanner stage 1 pubic hair and Tanner stage 1 breasts, cervix, and small uterus. What is the enzyme deficiency? (2007, 2011, 2012, 2018)
- CYP17
- CYP21
- 3-beta-hydroxysteroid dehydrogenase deficiency (3-beta-HSDD)
CYP17
- AR inheritance
- Chromosome 10
- Delayed puberty, primary amenorrhea, hyper hypo, HTN (increased mineralocorticoids)
A 16yo presents with primary amenorrhea, elevated FSH, and elevated progesterone 25ng/mL? What condition does she have? (2012, 2018)
- 45,X
- 46,XY, complete gonadal dysgenesis
- CYP21 deficiency
- CYP17 deficiency
CYP17 deficiency
Where is the CYP17A1 enzyme located? (2015)
- Mitochondria
- Endoplasmic Reticulum
Endoplasmic Reticulum
How is CYP21 inherited? (2018)
- Chromosome 6
- Autosomal Recessive (CYP21A1 pseudogene, CYP21A2 gene)
What is 17-hydroxylase deficiency (17-OHD) most associated with? (2020)
- Hyponatremia
- Hypotension
- Hyperkalemia
- Ambiguous genitalia
Ambiguous genitalia
List the CAH enzyme deficiencies in order of most common to least common. (2018)
- 21-hydroxylase (21-OH)
- 11-hydroxylase (11-OH)
- 3-beta-hydroxysteroid dehydrogenase (3-beta-HSD)
46,XX, ambiguous genitalia most common condition? (2018)
- Ovotesticular DSD (true hermaphroditism)
- Testicular DSD (sex reversal; de la Chapelle syndrome)
- CAH
CAH
(Image of an infant with a small phallus, opening on underside of phallus, scrotal mass on the right) Karyotype is 46,XX. What is the most likely diagnosis? (2014, 2015)
- Ovotesticular DSD (true hermaphroditism)
- 5-alpha-reductase deficiency
- Mixed gonadal dysgenesis
- AIS
- CAH
Ovotesticular DSD (true hermaphroditism)
(Image of an infant with a small phallus, opening on underside of phallus, scrotal mass on the right) Karyotype is 46,XY. What is the least likely diagnosis? (2014, 2015)
CAH
A baby is born with ambiguous genitalia including a phallus with a urogenital sinus at its base and a unilateral palpable inguinal mass. Karyotype 46,XX. What is the most likely diagnosis? (1999, 2003, 2010)
- CAH
- 46,XX male
- 5-alpha-reductase deficiency
- Mixed gonadal dysgenesis
46,XX male
What is least likely with an 11beta-hydroxylase (11-OH) deficiency mutation? (2014)
- Hypertension
- Hyperkalemia
- Hypernatremia
Hyperkalemia
All but which of the following will not result in ambiguous genitalia? (2012)
- CYP11 deficiency
- CYP17 deficiency
- CYP21 deficiency
- 3-beta-hydroxysteroid dehydrogenase deficiency (3-beta-HSDD)
CYP17 deficiency
How do you monitor a 14yo on glucocorticoid and mineralocorticoid replacement? (2007)
- Cortisol
- ACTH
- ACTH stimulation test
- ACTH after CRH stimulation test
- Renin/angiotensin
Renin/angiotensin
An adolescent girl with primary amenorrhea is found to have Mullerian agenesis and a 1cm vaginal dimple. Which is the most appropriate treatment when she is ready to become sexually active? (2003, 2010)
- Frank dilators
- Ingram procedure
- McIndoe procedure
- Vecchietti procedure
Frank dilators
What is the least common finding in Turner syndrome? (2003)
- Hypothyroidism
- Diabetes
- Fragment of Y chromosome
Fragment of Y chromosome
What excludes the diagnosis of 21-hydroxylase deficiency (21-OHD) in a patient with new onset hirsutism at puberty? (2005)
- ACTH stimulation test
- 17-hydroxyprogesterone (17-OHP)
17-hydroxyprogesterone (17-OHP)
A 22yo is 60in tall with a broad/wide/shield chest, breast development, and menstrual irregularities. What is the most likely diagnosis? (2005, 2007, 2023 NIH)
- 45,X
- 45,X/46,XX
- 45,X/46,XY
- 46,XX
45,X/46,XX
What is the most likely karyotype for a patient with gonadal dysgenesis? (2003, 2007)
- 45,X
- 45,X/46,XX
- 47,XYY
45,X/46,XX
What is the most likely phenotype in an AMH receptor defect? (2019)
- Persistence of Mullerian structures
- POI
- Precocious puberty
Persistence of Mullerian structures
How do you calculate the expected adult height of a child based on parental heights in centimeters? (2010)
Add height of both parents +/-5 depending on sex / 2
What is the correct order of pubertal events in girls? (2010, 2012, 2013, 2014, 2016, 2018)
- Maximal growth, thelarche, pubarche, menarche
- Thelarche, menarche, pubarche, maximal growth
- Thelarche, pubarche, maximal growth, menarche
- Pubarche, thelarche, maximal growth, menarche
Thelarche, pubarche, maximal growth, menarche
AG-BPM
Adrenarche
Growth spurt
Breast
Pubarche
*final growth spurt
Menarche
What is the first hormone to rise during adrenarche? (2011, 2018)
- DHEAS
- Androstenedione
- Testosterone
- Estradiol
- ACTH
What is the first hormone to rise during puberty? (2018)
- Estrogen
- LH
- FSH
- Androstenedione
LH
What is the change in LH seen as the first sign of puberty? (2009, 2012, 2018)
- LH in daytime
- LH in nighttime
- LH while asleep
LH in nighttime
What test do you order from isolated precocious adrenarche? (2011, 2018)
- DHEAS
- Androstenedione
- 17-hydroxyprogesterone (17-OHP)
- FSH
- E2
17-hydroxyprogesterone (17-OHP)
What is the first change seen in adrenarche? (2018)
- Increase in testosterone
- Increase in DHEA
- Increase in androstenedione
- Increase in ACTH proportional to reticularis growth
Increase in DHEA
What hormone is highest in adrenarche? (2018)
- DHEA
- Androstenedione
- Total testosterone
DHEA
What best describes adrenarche? (2011)
- Increased ACTH with enlargement of the zona reticularis
- Increased DHEAS at 6-7 yo
Increased DHEAS at 6-7 yo
What is the first sign of male puberty? (2007, 2018)
- Hair growth around testicles
- Testicle enlargement
- Phallus enlargement
Testicle enlargement
What is the most common presentation in the setting of an inactivated alpha subunit GPCR? (2021)
- Delayed puberty
- Precocious puberty
- Ambiguous genitalia in a male
- Ambiguous genitalia in a female
Delayed puberty
What is the most common cause of delayed puberty and primary amenorrhea in females? (2011, 2012, 2017, 2018, 2022)
- (Stress induced) Hypergonadotropic hypogonadism (hyper-hypo)
- Hypogonadotropic hypogonadism (hypo-hypo)
- MRKH
- Constitutional delay
- Cushing syndrome
- Craniopharyngioma (CNS tumors)
- Turner syndrome
- Prolactinoma
(Stress induced) Hypergonadotropic hypogonadism (hyper-hypo)
What is the most common cause of delayed puberty in females? (2015, 2021)
- Eugonadal/Constitutional
- Hypergonadotropic hypogonadism (hyper-hypo))
- Hypogonadotropic hypogonadism (hypo-hypo)
Hypergonadotropic hypogonadism (hyper-hypo))
What is the most common cause of primary amenorrhea? (2003, 2010, 2011, 2014, 2015, 2023 NIH)
- Hypergonadotrophic hypogonadism (hyper-hypo)
- Hypogonadotropic hypogonadism (hypo-hypo)
- Mullerian agenesis
- 46,XY, complete gonadal dysgenesis
- CAH
- AIS
- Hyperprolactinemia
Hypergonadotrophic hypogonadism (hyper-hypo) per 2003, 2010, 2014, 2023 NIH
A patient presents with anosmia and amenorrhea. Where is the defect most likely located? (2007, 2023 NIH)
KAL
What gene causes autosomal dominant Kallman syndrome? (2013, 2014, 2015, 2017, 2018, 2022, 2023 NIH)
- FGFR1
- KAL1
- KAL2
- PROK
- KISS
FGFR 1 = KAL2
What is the inheritance pattern of Kallmann syndrome with KAL1? (2021)
- X-linked recessive
- Autosomal dominant
- Autosomal recessive
X-linked recessive
All of the following are associated with the KAL1 gene except? (1999)
- Guides gonadotroph cell migration
- Is found on the distal end of the X chromosome
- Is sometimes associated with congenital short stature
- Is adjacent to region encoding for congenital ichthyosis
Is sometimes associated with congenital short stature
In a woman with Kallmann syndrome picture (hypo-hypo and anosmia) what is the most likely mutation? (2021)
- KAL1
- KAL2
- FGFR1
- PROK2
KAL2 = FGFR1
What is the most likely inheritance of KAL2? (2015, 2018)
- Autosomal dominant
- Autosomal recessive
- X-linked
Autosomal dominant
What is the most common transmission of Kallmann syndrome? (2011, 2012, 2013, 2018, 2023 NIH)
- X-linked
- Autosomal recessive
- Autosomal dominant
- Sporadic mutation
X-linked (anosmin, KAL1)
Kallmann syndrome is a disorder of what? (2005, 2018, 2023 NIH)
- Neuronal synthesis
- Neuronal migration
- Olfactory placode tumor
- GnRH synthesis
Neuronal migration
A 16yo F presents with delayed puberty. What is the best first test? (2021)
- FSH
- E2
- Testosterone
A 16yo F presents with delayed puberty. Thyroid testing and prolactin are normal. What is the best next test? (2003)
- FSH
- E2
- Testosterone
FSH
A 15yo F with male build, new onset clitoromegaly. Most likely diagnosis? (2019, 2021)
5-alpha-reductase deficiency
A 15yo presents with primary amenorrhea, has Tanner stage 1 breasts, clitoromegaly, no uterus, and a male body habitus. What test is most likely elevated? (2016, 2017, 2022)
- 17-hydroxyprogesterone (17-OHP)
- T/DHT ratio
- 11-deoxycorticosterone (11-DOC)
- FSH
- DHEAS/androstenedione ratio
T/DHT ratio (5 alpha reductase deficiency)
What are the internal and external structures of a patient who is 46,XY with 17-hydroxylase deficiency (17-OHD)? (2013, 2016, 2018)
- Internal male, external male
- Internal male, external female
- Internal female, external male
- Internal female, external female
Internal male, external female
A 5yoF with a pigmented lesion that does not cross the midline presents with vaginal bleeding. What is the most likely cause? (2011, 2021)
- GnRH pulsatility
- CAH
- Ovarian cysts
A 5yoF with skin spots that don’t cross the midline presents with precocious puberty and 90th percentile for height. What is the most likely cause? (2003, 2007)
- GnRH pulsatility
- CAH
- Ovarian cysts
Ovarian cysts (McCune Albright)
What is likely seen in a GNAS activating mutation? (2023 NIH)
What does an error in the GNAS gene lead to? (2018)
Increase in protein kinase A (PKA)
McCune-Albright syndrome
What is being overproduced in McCune-Albright syndrome? (2009, 2010, 2015)
- Protein kinase C (PKC)
- Protein kinase A (PKA)
- DAG
Protein kinase A (PKA)
What occurs in McCune-Albright syndrome? (2011)
- Cytochrome P450 enzyme activation
- Protein kinase A (PKA) activation
- Others
Protein kinase A (PKA) activation
What is the most effective treatment of McCune-Albright syndrome? (1999)
- GnRH analog
- GnRH agonist
- Testolactone
- Ketoconazole
- Spironolactone
- Testolactone is no longer available
- Use anastrozole, though loses effect over time
- GnRH agonist may work if GnRH independent > dependent
- Other: tamoxifen
A patient presents with café-au-lait spots, bone lesions, and precocious puberty. Which of the following is most likely to be activated? (2005)
Protein kinase A (PKA)
Cytochrome P450
Others
Protein kinase A (PKA)
A young girl presents with precocious puberty, café au lait spots, and polyostotic fibrous dysplasia. These findings are most likely caused by which of the following? (2003)
- Hypersecretion in multiple endocrine organs
- A lesion leading to constitutive activation of adenylate cyclase
A lesion leading to constitutive activation of adenylate cyclase
What is AMH in patients with hypogonadotropic hypogonadism (hypo-hypo)? (2020)
- High
- Low
- Normal
Normal
What is the most common autosomal dominant genetic defect for hypogonadotropic hypogonadism (hypo-hypo)? (2014, 2018, 2023 NIH)
- KAL1 mutation
- FGFR1 mutation
- Androgen receptor mutation
- SRY mutation
- AZF mutation
FGFR1 or KAL2 for Kallman
What is the best way to establish a diagnosis in a patient with hypogonadotropic hypogonadism (hypo-hypo) and anosmia? (2020)
- MRI
- Gene sequencing
Gene sequencing
A female patient has Kallman syndrome. What mutation is most likely to affect 50% of her offspring? (2019)
- KAL2
- FGFR2
KAL2 or FGFR1 (AD)
What hormone “turns on” the HPO axis in puberty? (2019)
- Kisspeptin
- Leptin
- GABA
Kisspeptin
What is the most important signal for the initiation of GnRH pulses to start puberty? (2023 NIH)
- Leptin
- Kisspeptin
Kisspeptin
Which substance is not responsible for the rise in GnRH with puberty? (2012, 2014, 2018)
- GABA
- Leptin
- Kisspeptin
- Endogenous opioids
- Glutamate
GABA
What will a mutation in the kisspeptin receptor most likely cause? (2015, 2023 NIH)
- Decreased LH/Failure of LH release
- Elevated TSH
- Elevated LH
- Elevated GnRH
Decreased LH/Failure of LH release
What is most likely associated with the onset of puberty? (2011, 2014, 2015, 2017, 2018)
- Kisspeptin
- Leptin
- Neuropeptide Y (NPY)
Kisspeptin
What is the most likely mechanism of precocious puberty at 6yo with pigmented skin lesions? (2019)
Constitutively active GPCR
What is the treatment for McCune-Albright syndrome? (2007, 2009, 2011, 2012, 2018, 2019)
- Aromatase inhibitor
- OCPs
- Progesterone
- Estradiol
Aromatase inhibitor
A patient presents with precocious puberty and is found to have bone and hyperpigmented skin lesions. What would you treat this patient with? (2003, 2007, 2009, 2011, 2012)
- Anastrozole
- Flutamide
- GnRH agonist
- Ketoconazole
- Progestin
- Testolactone
Testolactone (no longer available)
Anastrozole
A young female presents with advanced bone age. What is the single best test to determine activation of the HPO axis? (2015)
- LH
- Estrogen
LH (GnRH stim testing)
What is the most common cause of hypogonadotropic hypogonadism (hypo-hypo) boys? (2017)
- Constitutional delay
- Kallmann syndrome
Constitutional delay
What is the most common mutation in familial hypogonadotropic hypogonadism (hypo-hypo)? (2016, 2018)
Kisspeptin loss of function
A male patient presents with signs of hypogonadism. What is the best test for evaluation? (2015)
- Serum total testosterone
- Serum total free testosterone
- FSH
- LH
Serum total testosterone
What is the most common cause of GnRH dependent precocious puberty? (2015)
- Idiopathic
- McCune-Albright syndrome
Idiopathic
A patient presents with precocious puberty and elevated gonadotropins. How is this treated? (2018)
- Aromatase inhibitors
- Danazol
- GnRH antagonists, daily preparations
- GnRH agonists, depot preparations
- GnRH agonists, depot preparations
This is GnRH-dependent precocious puberty (McCune Albright has low gonadotropins due to elevated sex steroids)
What are the indications for treatment of precocious puberty? (2018)
- Thelarche
- Pubarche
- Progression to the next stage in 3-6 months
- Current isolation from friends
Progression to the next stage in 3-6 months
A 5yoF presents with precocious puberty and an ovarian cyst. What is the most likely diagnosis? (2014, 2018)
- Granulosa cell tumor
- Theca cell tumor (thecoma)
Granulosa cell tumor
Which of the following can be used to monitor for recurrence of a granulosa cell tumor? (2003)
- Inhibin B
Inhibin B
What is the best tumor marker for a granulosa cell tumor? (2003, 2005, 2023 NIH)
- Estradiol
- Inhibin B
- AFP
- hCG
- CA125
Inhibin B
What is the most likely diagnosis of a patient presenting with precocious puberty and delayed bone age? (2009, 2013, 2014, 2015, 2016, 2017)
- Hypothyroidism
- McCune-Albright syndrome
Hypothyroidism
A patient presents with isolated precocious pubarche. What lab should be ordered next? (2007)
17-hydroxyprogesterone (17-OHP)
What is the mechanism of action of McCune-Albright syndrome? (2009, 2012, 2015, 2016, 2018)
- Constitutive protein kinase A (PKA)
- Phospholipase C (PLC)
- IP3/DAG
Constitutive protein kinase A (PKA)
What is the mechanism of action of McCune-Albright syndrome? (2011, 2012)
- G inhibitory stimulation
- GsAlpha stimulation
- GPR40 mutation
- Gq stimulation
- PLC
GsAlpha stimulation
What is responsible for McCune-Albright syndrome symptoms? (2009, 2012)
- Increased cytochrome P450 enzyme activities
- Increased protein kinase A (PKA)
- Decreased protein kinase C (PKC)
Increased protein kinase A (PKA)
What is the mechanism of McCune-Albright syndrome? (2007, 2018)
- Inactivating mutation of the LH receptor
- Activating mutation of the beta subunit of the G-protein
- Constitutively active adenylate cyclase due to somatic mutation alpha subunit of the G-protein GNAS1 gene
- Inactivating mutation of the alpha subunit of the G-protein
Constitutively active adenylate cyclase due to somatic mutation alpha subunit of the G-protein GNAS1 gene
What is the best (most sensitive) test for growth hormone deficiency? (2014, 2015, 2017, 2023 NIH)
- Insulin tolerance test (ITT)
- Glucose stimulation test
- IGF-1 level
IGF-1 level
What is the best hormonal evaluation in puberty? (2016)
- LH:FSH ratio <1 in a prepubertal patient
- LH <1 during GnRH stimulation test
- AMH undetectable in a prepubertal patient
- Inhibin undetectable in a prepubertal patient
LH <1 during GnRH stimulation test
What does the presence of breasts indicate? (2016, 2018)
- Estrogen exposure at some point
- Testosterone
Estrogen exposure at some point
Children with sexual precocious puberty at most risk for which of the following? (2016)
- Sexual molestation
- Depression and behavior disorders
- Short stature
Depression and behavior disorders
Why has the age at puberty decreased in the United States? (2014, 2015)
- Increased health
- Increased fat mass
Increased fat mass
How do you give HRT to patients with delayed puberty? (2015, 2016, 2017)
- E2 slowly increase over 1-2yr
- E2 and P4
- E2 plus growth hormone
E2 slowly increase over 1-2 yr
What condition can be seen as virilization at puberty? (2005, 2007, 2009, 2011, 2012, 2018)
- 5-alpha-reductase deficiency
- Complete AIS
- 46,XY complete gonadal dysgenesis (Swyer syndrome)
- 17-beta-hydroxylase deficiency (17-beta-OHD)
- FSH activating receptor
5-alpha-reductase deficiency
What adrenal enzyme deficiency is least likely to cause virilization at puberty? (2011)
- 17-beta-hydroxysteroid dehydrogenase deficiency (17-beta-HSD)
- 17-hydroxylase deficiency (17-OHD)
- Aromatase deficiency
17-hydroxylase deficiency (17-OHD)
A 14 yo presents with primary amenorrhea, Tanner stage 4 pubic hair, Tanner stage 5 breasts, 2 cm blind vagina with dimple, normal hymen, no bulging, and no pain. What is the most likely diagnosis? (2012, 2018)
- Androgen insensitivity
- Imperforate hymen
- Transverse vaginal septum
- Mullerian aplasia
Mullerian aplasia
A 14yo presents with amenorrhea, Tanner stage 4 breasts, Tanner stage 5 pubic hair, 2cm vaginal dimple, no bulge, no pain, and 63in height. What is the most likely diagnosis? (2023 NIH)
- Mullerian anomaly
- AIS
- Imperforate hymen
Mullerian anomaly
A 14 yo presents with primary amenorrhea, Tanner stage 4 breasts, 2cm vaginal dimple, no bulge, no pain, and 63in height. What is the best test to make the diagnosis? (2023 NIH)
- MRI
- Serum testosterone level
Serum testosterone (to distinguish from AIS)
A 17yo presents with amenorrhea, Tanner stage 4 breasts, Tanner stage 5 pubic hair, normal external genitalia, intact hymen, and 2cm vaginal pouch (vaginal dimple). What is the best next step? (2011, 2012, 2018, 2023 NIH)
- Vaginoscopy
- Exam under anesthesia
- Pelvic ultrasound (US)
- CT scan of pelvis
Pelvic ultrasound (US)
A patient presents with labs consistent with hypogonadotropic hypogonadism (hypo-hypo). What is the least likely cause? (2009)
- DAX1
- FMR1
- Leptin
FMR1
An 18 yo 46,XY female presents with new onset breast (and pubic hair) development. What is the most likely cause? (2009, 2011, 2018)
- Dysgerminoma
- Gonadoblastoma
- Activating LH mutation
- Sertoli-Leydig cell tumor
- Seminoma
- Granulosa cell tumor
- Theca-lutein cyst
Gonadoblastoma
A 30yo 46,XY female presents with new onset breast and pubic hair development. What is the most likely diagnosis? (2011)
- Gonadoblastoma
- Seminoma
- Granulosa cell tumor
Gonadoblastoma
What is the most likely cause of cancer before 30yo in a phenotypic female with a genotype of 47,XY? (2009)
- Mixed gonadal dysgenesis
- 46,XY, complete gonadal dysgenesis (Swyer syndrome)
- 46,XX, ovotesticular DSD (true hermaphroditism)
- 46,XY, gonadal dysgenesis
- Complete AIS
- Incomplete AIS
46,XY, complete gonadal dysgenesis (Swyer syndrome)
Which of the following is the most likely to develop a gonadoblastoma (before 30yo)? (2005, 2011)
- Mixed gonadal dysgenesis
- 46,XX, ovotesticular DSD (true hermaphroditism)
- 46,XY, gonadal dysgenesis
- Complete AIS
- Incomplete AIS
46,XY, gonadal dysgenesis
Which of the following is the most likely to develop gonadal cancer? (2009)
- 46,XY complete gonadal dysgenesis (Swyer syndrome)
- Mixed gonadal dysgenesis
- AIS
- 45,X
46,XY complete gonadal dysgenesis (Swyer syndrome)
What is the least important in the work-up of small testes? (2018)
- Karyotype
- Prolactin
- LH/FSH
- Testosterone
- DHEAS
DHEAS
A patient presents with labs consistent with hypogonadotropic hypogonadism (hypo-hypo). What is the most common cause? (2011)
- Increased endogenous opioids leading to GnRH suppression
- Hyperprolactinemia
- Increased cortisol
Hyperprolactinemia
A 2yo has a bone age of 4.5 yo and breast development. What is the best test to determine if there has been premature activation of the HPO axis? (2014, 2015)
- LH
- GnRH
- Estradiol
LH
A young girl presents with vaginal bleeding, Tanner stage 3 breasts, and 90th percentile for height and weight. What is the most likely diagnosis? (2011, 2014, 2015)
- Exogenous hormone ingestion
- Ovarian cyst
Ovarian cyst
A 23 yo F presents with new onset of hypogonadotropic hypogonadism (hypo-hypo). What is the most likely etiology? (2011, 2015)
- Pituitary tumor
- Hyperprolactinemia
- Hypothalamic tumor
- Idiopathic
Hyperprolactinemia
A woman is found to have hypogonadotropic hypogonadism (hypo-hypo) with a normal prolactin and TSH. What do you do next? (2011, 2012, 2023 NIH)
- GnRH stimulation test
- MRI
- ACTH stimulation test
MRI
A woman recently finished tuberculosis treatment and now has weight loss, hyper-pigmented lesions, and fatigue. What is the best next test? (2011)
- 24hr urine free cortisol
- ACTH stimulation test
- Dexamethasone suppression test
ACTH stimulation test (for adrenal insufficiency)
A patient recently had transsphenoidal surgery for a growth hormone tumor. She now has weight loss, hyperpigmented lesions, and fatigue with a normal TSH and ACTH. What do you do next? (2009)
- IGF with cortisol
- ACTH stimulation test
- Dexamethasone suppression test
ACTH stimulation test
A patient presents with bilateral adrenal cysts and an elevated ACTH after high dose dexamethasone suppression test. What is the most likely diagnosis? (2011)
- Pituitary tumor
- Adrenal tumor
- Ectopic ACTH production
Ectopic ACTH production
What is the most common location of ectopic ACTH production? (2009)
- Lung
- Liver
- Ovary
- Kidney
Lung
A (receptor inactivating) mutation in which of the following is most likely to have increased ovarian androgen production? (2009, 2023 NIH)
- Insulin receptor
- IGF-1 receptor
- Leptin
- 21-hydroxylase (21-OH)
Insulin receptor
(IGF-1 promotes androgen production, Leptin inhibits, 21-hydroxylase does not exist in ovary)
Primary pathology in women with HAIR-AN and hyperthecosis is severe insulin resistance, resulting in grossly elevated insulin levels that stimulate ovarian androgen production in theca cells (via insulin, IGF-1, and hybrid receptors) and markedly decrease hepatic SHBG production, thereby greatly increasing the amount of circulating free androgen.
What causes abnormal puberty? (2009)
- Craniopharyngioma
- Prolactinoma
- Hamartoma
Hamartoma
Craniopharyngioma > hypo hypo > delayed puberty
Prolactinoma > hypo hypo > delayed puberty
Hypothalamic hamartomas, congenital malformations composed of a heterotropic mass of nerve tissue usually located on the floor of the 3rd ventricle or attached to the tuber cinereum, are a common etiology of precocious gonadarche.
An 18yoF comes to the office prior to going to college. While she has had normal breast development, she has never had a period. She has been a swimmer throughout middle and high school and was told this prevented her periods from starting. Given she swims every day, she hasn’t cared about not having a period but her mother wants this evaluated prior to her starting college. Which of the following is least likely to be the etiology of her amenorrhea? (2022)
- Hypothalamic dysfunction
- POI
- Mullerian agenesis
- AIS
- Delayed puberty
Delayed puberty
A physical exam on the 18yoF with primary amenorrhea after being a competitive swimmer through middle and high school reveals normal breast development but no uterus. Which lab evaluations would be the most appropriate next step? (2022)
- FSH, LH, estradiol
- FSH, TSH, prolactin
- TSH, prolactin, testosterone
- Testosterone, karyotype
Testosterone, karyotype
A 30yoF comes to the office to begin treatment for infertility. She had menarche at 14yo and regular menstrual cycles until starting OCPs at 21yo. She has continued OCPs until 6mo ago and has not had menses since. What is the least likely diagnosis? (2022)
- Post-pill amenorrhea
- POI
- PCOS
- Hypothyroidism
- Hyperprolactinemia
Post-pill amenorrhea
In a patient with amenorrhea, presence of breasts and a uterus, with a normal laboratory evaluation (FSH, TSH, prolactin), which of the following would be most helpful in determining the etiology? (2022)
- Presence of PCOS on ultrasound (US)
- Estrogen status
- History of bleeding while on OCPs
- Patient weight
Estrogen status
A 17yoF presents with normal thelarche and adrenarche and primary amenorrhea. What will serum estradiol levels most likely reveal? (2022)
- Elevated levels of estradiol due to tissue resistance
- Very low (menopausal) levels of estradiol
- Levels in the normal range for a 17yoF
- Undetectable levels due to inhibitor antibodies
Levels in the normal range for a 17 yo F (Mullerian agenesis)
A 17 yo F presents with normal thelarche and adrenarche and primary amenorrhea. What will serum testosterone levels most likely reveal? (2022)
- Levels in the normal range for a 17yoF
- Levels in the normal range for a 17yoM
- Elevated levels due to mutation of the androgen receptor
- Low levels due to increased aromatase
- Low levels due to gonadal failure
Levels in the normal range for a 17 yo F (Mullerian agenesis)
A 17yo presents with primary amenorrhea, normal secondary sexual characteristics, and a blind vaginal pouch on pelvic exam. What is the most appropriate next step? (2022)
- Office cystoscopy
- CT scan of the pelvis
- Serum testosterone
- MRI of the olfactory bulbs
- Office hysteroscopy
Serum testosterone (to distinguish between AIS and Mullerian anomaly)
A 17 yo presents with primary amenorrhea, normal secondary sexual characteristics, a blind vaginal pouch, and a uterus but no cervical tissue on MRI. What is the most appropriate surgical procedure? (2022)
- Laparoscopic resection of the non-functioning uterine remnants
- Robotic resection of the non-functioning uterine remnants
- Laparoscopic resection of all uterine and ovarian structures to avoid neoplasia
- Laparoscopic resection of any functional uterine remnants
Laparoscopic resection of any functional uterine remnants
What is the first steroid hormone to increase in a pubertal girl? (2003)
DHEAS
17-beta-estradiol
DHEAS
Which is least likely to be seen in the ovaries of a prepubertal girl? (2003)
- Antral follicles
- Corpus albicans
Corpus albicans
A 15yo F presents with primary amenorrhea and is phenotypically prepubertal on exam. What do you evaluate first? (2003, 2023 NIH)
- FSH
- Prolactin
- MRI
- Bone age
Bone age
A 6yoF has growth hormone deficiency. She would be most expected to have which of the following additional consequences (2003)
Pubertal delay
A 16 yo F has primary amenorrhea and anosmia. What is the most likely cause of her problem? (2003)
Mutation of the X chromosome Interfering with migration of neurons
Which null mutation is most likely to result in hypergonadotropic hypogonadism (hyper-hypo)? (2005)
- LH receptor
- FSH receptor
LH receptor
A young patient presents with Tanner stage 1 adrenarche, Tanner stage 3 breasts, and a blind vagina. What is the most likely genetic defect? (2005)
- AMH
- Androgen receptor
androgen receptor
18yo boy presents with delayed puberty and hypothalamic hypogonadism. What intra-cellular effect would likely be found in his GnRH neurons? (2024)
- Increased cAMP activity
- Decreased Ca
- Decreased protein kinase A (PKA)
- Decreased tyrosine phosphorylation
Decreased Ca
A 5 yo African American girl presents with has pubic hair, Tanner stage 1 breasts, and growth 60th percentile. What do you do next? (2003, 2012, 2021)
- Estradiol level
- DHEA
- Testosterone
- Bone age
Bone age
What is the least likely to cause amenorrhea? (2023 NIH)
- Hypothyroid
- Elevated prolactin
- PCOS
- Hypopituitarism
Hypothyroid
What percentage of adolescents with POI and secondary amenorrhea have an abnormal karyotype? (2023 NIH)
- 50%
- 13%
13%
Patients with Sheehan syndrome or pituitary apoplexy are most likely to have which hormone is affected? (2018, 2022)
- FSH/LH
- ACTH
- Growth hormone
- TSH
GH
*Anterior pituitary defect: Prolactin, GH, and GnRH deficiencies are most common with Sheehan, but ACTH and TSH also possible.
Oxytocin and vasopressin (ADH) are not compromised
Sheehan syndrome least associated with deficiency in? (2011, 2018, 2023 NIH)
- Oxytocin
- Growth hormone
- Prolactin
- ACTH
- TSH
- Estrogen
- FSH
- LH
Oxytocin
What is least likely to be affected in Sheehan syndrome? (2011)
- Growth hormone
- Prolactin
- TSH
- ACTH
TSH
Most common deficiencies in Sheehan: GH, prolactin
Less common deficiencies: ACTH > TSH (corticotrophs are more sensitive to ischemia)
What is the first sign of postpartum pituitary apoplexy? (2013, 2015)
Severe headaches
How will a woman with Sheehan syndrome present? (2015)
- Decreased breastfeeding with little pubic hair
- Decreased breastfeeding with hypercalcemia
Decreased breastfeeding with little pubic hair
*Sheehan a/w hyponatremia, hypokalemia, hypocalcemia, hypomagnesemia, and hypophosphatemia
How will a woman with Sheehan syndrome present? (2015)
- Galactorrhea and hypermenorrhea
- Amenorrhea, difficulty nursing, and hypercalcemia
- Amenorrhea, difficulty nursing, and decreased pubic hair
Amenorrhea, difficulty nursing, and decreased pubic hair
Described large postpartum hemorrhage. What would you expect to see as a consequence? (2023 NIH)
- Failed lactation
- Failed nursing, amenorrhea, loss of pubic hair
- Failed nursing alone
- Galactorrhea, amenorrhea, HTN
Failed nursing, amenorrhea, loss of pubic hair
What is least likely to cause bilateral hemianopsia 3wk postpartum? (2003, 2010, 2012, 2023 NIH)
- Sheehan syndrome
- Lymphocytic hypophysitis
- Rathke pouch cyst
- Craniopharyngioma
Sheehan syndrome
A 32wk pregnant patient with prolactinoma and visual changes, how do you treat? (2014, 2019, 2021)
- Bromocriptine
- Transsphenoidal surgery
- Deliver then transsphenoidal surgery
Bromocriptine
A 32wk pregnant woman gestation with a known 12mm prolactinoma from before pregnancy. Her prolactin is 35ng/dL and she now has headache and bitemporal hemianopsia. What is the best course of action? (2014)
- Delivery and then transsphenoidal surgery
- Transsphenoidal surgery
- Bromocriptine
Bromocriptine
When you give bromocriptine to a pregnant woman what happens to prolactin? (2003, 2005, 2009, 2011, 2018, 2023 NIH)
- Increased maternal prolactin
- Increased fetal prolactin
- Decreased fetal prolactin
- Altered/Decreased in amniotic fluid
- Decreased maternal TSH
- Decreased placental prolactin
Decreased fetal (and maternal) prolactin
(Amniotic fluid prolactin is not altered)
What is the most common side effect of cabergoline? (2021, 2022, 2023 NIH)
- GI upset
- Headache
- Cardiac valvulopathy
GI upset
What steroid is most likely to be increased with elevated prolactin? (2005, 2014, 2015)
- DHEAS
- Androstenedione
DHEAS
*Some women with hirsutism also have mild hyperprolactinemia. Elevated serum prolactin concentrations can be associated with increased serum DHEA-S levels. Prolactin receptors have been identified in the human adrenal, and prolactin can increase adrenal DHEA production in vitro. Although DHEA-S is a weak androgen, it can be converted in the periphery to potent androgens such as testosterone and DHT. Prolactin stimulation of adrenal androgen production, excessive ovarian androgen production due to the chronic anovulation that often results from hyperprolactinemia (discussed in Chapter 4), and increased free testosterone levels consequent to anovulation may all be relevant to causation of hirsutism in women with chronic hyperprolactinemia.
An increased prolactin is associated with which of the following? (2019, 2020)
- DHT
- Testosterone
- Androstenedione
- DHEA
- DHEAS
DHEA-S
What do you check next in a patient who has prolactin 30ng/dL (upper limit of normal 24ng/dL)? (2021)
- Repeat prolactin
- TSH
- FSH
- LH
Repeat prolactin
What hormone is most likely to be also elevated with a prolactinoma? (1999, 2003, 2005, 2007, 2009, 2010, 2012, 2019, 2021)
- Cortisol
- FSH
- Inhibin
- Growth hormone
- ACTH
- T4
- TRH
Growth hormone
What hormone is most likely to be affected after a pituitary infarction? (2016, 2019, 2021)
- Growth hormone
- TSH
- ACTH
- FSH
- Gonadotropins
GH
What type of WBC most commonly infiltrates the ovary in autoimmune oophoritis? (2021)
- Lymphocyte
- Neutrophil
- Eosinophil
- Macrophage
Lymphocyte
What is the most important test in suspected autoimmune oophoritis? (2014, 2015)
- Anti-21-hydroxylase (21-OH) antibodies
- Antithyroid antibodies
- Antiovarian antibodies
Anti-21-hydroxylase (21-OH) antibodies
Most important test in suspected autoimmune oophoritis? (2023 NIH)
- Adrenal antibodies
- Ovarian antibodies
- Thyroid antibodies
Adrenal antibodies
What antibody is most highly associated with POI? (2003, 2005, 2007, 2009, 2010, 2014, 2018, 2023 NIH)
- Antithyroid antibodies
- Antiadrenal antibodies
- Antiovarian antibodies
- Antinuclear antibodies
Anti-thyroid abs (more common)
Anti-adrenal abs (help confirm diagnosis)
Having a positive finding of which of the following antibodies would be most related to autoimmune POI? (2011, 2012)
- Antithyroid antibodies
- Antiadrenal antibodies
- Antiovarian antibodies
Anti-adrenal antibodies
The most likely cause of POI? (2021)
- 45,X
- 45,X/46,XX
- FGFR1
- FMR1
45,X/46,XX
What is most likely to be found on evaluation of a 22 yo F with POI? (2014, 2015)
- Increased number of CGG repeats in FMR1
- Antiovarian antibodies
- Karyotype 46,XX
- Karyotype 46,X
Karyotype 46,XX
*Idiopathic POI is most common
A 22 yo F presents with POI. What is the most likely cause? (2014, 2020)
- FOXL2 mutation
- FMR1 premutation
- FSH receptor mutation
- LH receptor mutation
FMR1 pre-mutation
What is the most common abnormality with POI? (2007, 2011, 2023 NIH)
- Adrenal crisis
- Hypothyroidism
- Insulin resistance
- Ovarian antibodies
Hypothyroidism
What is most likely to be abnormal in a female with galactosemia? (2012, 2014, 2015, 2018, 2019)
- AMH
- TSH
- Prolactin
- Glucose
AMH
Galactosemia:
- Very rare
- Autosomal recessive
- Deficiency of galactose 1-phosphate uridyl transferase
- Toxicity of galactose metabolites in primordial follicles (decreased)
A 30 yo F presents with galactosemia. What else is she at risk for? (2023 NIH)
POI
What is the least likely cause of amenorrhea? (2013, 2014, 2015, 2023 NIH)
- Swimming
- Biking
- Running
- Ballet
- Gymnastics
Swimming
What is the most likely to cause anovulation? (2013)
PCOS
What is the most likely to cause anovulation? (2011)
- Hyperprolactinemia
- Hyperandrogenism
- Hypogonadotropic hypogonadism (hypo-hypo)
- Hypothyroidism
Hyperprolactinemia
What hormone do you measure in a patient with a prolactin secreting macroadenoma? (2013, 2018, 2023 NIH)
- TSH
- Cortisol
- IGF-1
- Alpha subunit
- FSH
IGF-1
What hormone level is most important to check after transsphenoidal surgery for a macroadenoma? (2016, 2022)
- Cortisol
- ACTH
- TSH
- FSH
Cortisol
Which test is least useful in evaluation of a macroadenoma? (2009, 2018, 2023 NIH)
- Prolactin
- FSH
- Alpha subunit
- IGF-1
- Cortisol
FSH
Gonadotrophs may secrete alpha-subunit instead of undetectable FSH
What do you expect to happen if polyethylene glycol (PEG) is added to a pretreatment serum sample of a patient with macroprolactinemia? (2023 NIH)
- Monomer
- Monomer <40%
- Polyethylene glycol (PEG) not the appropriate test
- Others
Precipitation of macroprolactin to allow appropriate measurement of free prolactin
A G2P0020 presents following two D&Cs with new onset hypomenorrhea. TVUS with a sagittal view of the uterus shows adhesions. What is the best next step? (2023 NIH)
- HSG
- Hysteroscopy
- Serum hCG
Hysteroscopy
How do you diagnose macroprolactinemia? (2023 NIH)
Pretreat the serum with polyethylene glycol (PEG)
A woman has experienced 9 mo of amenorrhea while nursing. What is the most likely diagnosis? (2005, 2023 NIH)
- Lactational amenorrhea
- Sheehan syndrome
- Prolactinoma
Lactational amenorrhea
What is the most likely karyotype in a patient with Turner syndrome presenting with secondary amenorrhea? (2005)
- 45,X
- 46,XX/45,X
46,XX/45,X
What antibody is most associated with POI? (2013, 2014, 2017)
- 21-hydroxylase (21-OH)
- Anti-TPO
Anti-TPO (commonly associated but not diagnostic)
21-hydroxylase (diagnostic)
What is most diagnostic of immune mediated POI? (2018)
- 21-hydroxylase (21-OH)
21-hydroxylase (21-OH)
A woman is found to have a mutation in a gene. What gene would her father need to be concerned about in terms of delayed disease? (2011)
- FMR1
- CFTR
- NCQV
- GnRH
FMR1
How many repeats are in Fragile X premutations? (2017)
- 5-36
- 38-54
- 55-200
- > 200
55-200
A woman has a brother with Fragile X syndrome. She has 100 CGG repeats on testing. What are the chances of her having POI? (2018)
- 0%
- 10%
- 20%
- 50%
- 100%
If a woman has a son or brother with mental retardation and negative work-up for Fragile X but she herself has 100 CGG repeats on FMR1 testing. What are the chances of her having POI? (2007, 2011)
- 10%
- 20%
- 40%
- 90-100%
20%
15% have pre-mutations in familial POI
1-7% have pre-mutations in sporadic POI
25 yo F has a son with mental retardation and a negative work-up for Fragile X. She gets screened and has 100 triplet repeats. What does this mean clinically? (2007, 2023 NIH)
- The son does have Fragile X
- The patient is at risk for POI
The patient is at risk for POI
A female patient is found to have a Fragile X pre-mutation. Which of the following is most likely to be true? (2011)
- 20% risk of POI
- Mental retardation
- Skeletal abnormalities
- Renal aplasia
20% risk of POI
What is the least common product in a hormonally active pituitary adenoma? (2014, 2015, 2023 NIH)
- Growth hormone
- ACTH
- Alpha subunit
- TSH
TSH
What genes are not involved with pituitary development? (2015)
- PIT-1
- PROP-1
- HESX1
- PTX (LHX)
- BMP15
BMP15
What gene is most associated with pituitary development? (2016, 2017)
- PIT-1
- PROP-1
- HESX1
- PTX (LHX)
- BMP15
PROP-1
What is the least likely congenital malformation in pituitary disease (2015)
- PROP-1
- PIT-1
- HOX genes
- BMP15
- Others
BMP15
What is most associated with POI/gonadal failure? (2003, 2014, 2015, 2016, 2017, 2018, 2023 NIH)
- Galactosemia
- Blepharophimosis inversus syndrome (FOXL2)
Galactosemia
A woman presents with elevated FSH, normal LH, increased alpha subunit, and a pituitary mass. What is the diagnosis? (2018)
- Prolactinoma
- TSH secreting adenoma
- Non-functional pituitary tumor
- FSH secreting adenoma
Non-functional pituitary tumor
80-90% of nonfunctional pituitary tumors derive from gonadotrophs.
FSH tumors are rare (high FSH, low LH)
Gonadotroph tumors may secrete large amounts of the a subunit
Gonadotroph tumors affect HPO axis via stalk compression
- interrupting GnRH
- blocking dopamine > increasing prolactin
LH/FSH secreted by same cell
Common a subunit (LH, FSH, TSH, hCG), different b subunit
GnRH stimulates common a and different b subunits vi g protein second messenger, calcium dependent system
Rate limiting step is synthesis of b subunits of each gonadotropin (prior question; remember, alpha subunits are common and secreted with non-functional tumors)
A patient presents in the third trimester of pregnancy with a new onset headache and prolactin 126ng/mL. What is the next step? (2005, 2012, 2018)
- Bromocriptine
- Neurosurgery consult
- MRI brain
- TSH level
MRI brain
Which is least likely to cause hypothalamic amenorrhea? (2003)
- Weight training
- Aerobic exercise
- Eating disorders
Weight training
A 25yoF is 9mo postpartum and breastfeeding. She has not had a menstrual cycle since before pregnancy. What is the least likely cause of her secondary amenorrhea? (2003)
- POI
- Sheehan syndrome
Sheehan syndrome
Which of the following is most likely to lead to amenorrhea with long term use? (2003)
- Fluoxetine
- Chlorpromazine
Chlorpromazine (anti-psychotic causing hyperprolactinemia)
A 30yoF presents with 3mo amenorrhea, FSH 35mIU/mL, LH 3mIU/mL, prolactin 15ng/mL, TSH 4mIU/L, and excess alpha subunit. What is the most likely diagnosis? (2007, 2023 NIH)
- POI
- Gonadotroph adenoma
Gonadotroph adenoma
How is bleeding induced in secondary amenorrhea? (2010)
Medroxyprogesterone (Provera)
A 37yoF presents with secondary amenorrhea for 3mo. She is found to have FSH 3mIU/mL, LH 54mIU/mL, and prolactin 18ng/mL. What do you check next? (2007
Pregnancy test
Which substance inhibits prolactin? (2018, 2023 NIH)
- GABA***
- TRH
- Glutamate
- VIP
- Angiotensin II
- Estrogen
GABA
Which of the following will increase prolactin? (2005, 2007, 2023 NIH)
- GABA
- CRH
- Angiotensin II per 2023 NIH
- Neuropeptide Y (NPY)
- Vasopressin
Angiotensin II
Which of the following most inhibits (most potent inhibitor of) prolactin release? (2011, 2012, 2014, 2015, 2017, 2018, 2019, 2023 NIH)
- VIP
- Angiotensin
- TRH
- Oxytocin
- GABA
GABA
A 30yoF has secondary amenorrhea after using OCPs for the past 9yrs and discontinuing them 6mo ago. Her exam revealed the presence of a normal uterus and breasts but low local estrogen in the vagina. Which lab evaluations would be the most appropriate diagnostic evaluation? (2022)
- FSH, LH, estradiol
- FSH, TSH, prolactin
- TSH, prolactin, testosterone
- Testosterone, karyotype
FSH, TSH, prolactin
A 30yoF has secondary amenorrhea after using OCPs for the past 9 yrs and discontinuing them 6mo ago. Her exam revealed the presence of a normal uterus and breasts but low local estrogen in the vagina. Labs demonstrate FSH 68mIU/mL, TSH 5.2mIU/L, and prolactin 2.5 ng/mL. What is the most appropriate next step? (2022)
- Repeat FSH and TSH in 4 wk
- Pituitary MRI
- Begin treatment of hypothyroidism
- Discuss oocyte donation
Repeat FSH and TSH in 4 weeks
A 30yo F has secondary amenorrhea after using OCPs for the past 9 yrs and discontinuing them 6 mo ago. Her exam revealed the presence of a normal uterus and breasts but low local estrogen in the vagina. Labs demonstrate FSH 68mIU/mL, TSH 5.2mIU/L, and prolactin 2.5ng/mL. Repeat FSH 4wk later is 75mIU/mL.
Which of the following is inappropriate to perform at this time? (2022)
- Antiadrenal and antiadrenal antibodies
- TSH, free T4, calcium, phosphate, CBC, fasting blood sugar
- Fragile X screening
- Ovarian biopsy
Ovarian biopsy
A 30yoF comes to the office to begin treatment for infertility. She had menarche at 14yo and regular cycles until starting OCPs at 21yo. She continued OCPs until 6mo prior to today’s visit. She has had no menses since. Lab evaluations reveal FSH 2.5mIU/mL, TSH 13mIU/L, and prolactin 52ug/L. What is the most appropriate next step in the evaluation? (2022)
- Repeat prolactin mid-morning
- Pituitary MRI
- Medroxyprogesterone acetate (MPA) (Provera) withdrawal
- Treatment with thyroid replacement
- Treatment with bromocriptine
Treat with thyroid replacement
Which of the following has the lowest risk for a young patient with POI treated with estrogen replacement, compared to women her age who are still cycling regularly? (2022)
- Breast cancer
- Endometrial cancer with unopposed estrogen
- Infertility
- Depression
Breast cancer
What is the best treatment for an infertile patient with hypothalamic amenorrhea with normal prolactin levels? (2003)
- Bromocriptine
- Clomiphene citrate (Clomid)
- hMG
hMG
What is the most common cause of oligo-ovulation? (2016, 2023 NIH)
- Weight loss/anorexia
- Hypothyroidism
- Hyperprolactinemia
Hyperprolactinemia
A 24 yo F with PCOS and a BMI 24 kg/m2 wants to get pregnant. What is the best way to do this? (2007, 2011)
- Clomiphene citrate (Clomid)
- Clomiphene citrate (Clomid) and metformin
- Metformin
- IVF
Clomiphene citrate (Clomid)
A 30 yo F with PCOS has infertility. What is the most appropriate therapy? (2011)
- Gonadotropins
- Metformin
- IVF
- Clomiphene citrate (Clomid)
- Letrozole
Letrozole
A woman with PCOS desiring fertility, fails clomiphene citrate (Clomid) therapy. What is the best next step? (2018)
- Ovarian drilling
- Gonadotropins
- Clomiphene citrate (Clomid) and -metformin
- Extended clomiphene citrate (Clomid)
Clomid + metformin
What test is least accurate for assessing insulin resistance? (2007)
- Fasting glucose:fasting insulin ratio
- HbA1c
- Euglycemic clamp
- 2hr oral glucose tolerance test (GTT)
- Fasting glucose
Fasting glucose
What is the best test to screen for insulin resistance? (2005, 2018)
- Frequently sampled IV glucose tolerance test
- Insulin tolerance test (ITT)
- Oral glucose tolerance test (GTT)
- HOMA-IR
- QUICKI
OGTT
What is the best/most accurate clinical test for glucose tolerance? (2003, 2005, 2010, 2011)
- Euglycemic clamp
- 2 hr oral glucose tolerance test (GTT)
- Fasting insulin:glucose ratio
Euglycemic clamp
What is the most accurate way to diagnose insulin resistance in PCOS? (2011)
- Insulin:glucose ratio
- HbA1c
- Oral glucose tolerance test (GTT)
- Fasting insulin
OGTT
What is the best test to evaluate glucose intolerance in PCOS? (2011)
- Fasting glucose
- 1hr glucose tolerance test (GTT)
- 2hr glucose tolerance test (GTT)
- HbA1c
2 hour GTT
What is the best screening for insulin resistance in PCOS? (2023 NIH)
- 2hr oral glucose tolerance test (GTT)
- Fasting glucose:insulin ratio
- HbA1c
2 hour OGTT
What causes increased insulin resistance in theca cells in women with PCOS? (2012, 2018)
- Decreased serine phosphorylation of insulin receptor
- Increased serine phosphorylation of the insulin receptor
- Increased tyrosine phosphorylation of the insulin receptor
- Decreased number of insulin receptors
Increased serine phosphorylation of the insulin receptor
What causes increased insulin with insulin resistance? (2012, 2014, 2015)
- Decreased number of insulin receptors
- Abnormality in insulin receptor
- Serine phosphorylation of insulin receptor cascade
Serine phosphorylation of insulin receptor cascade
What is the mechanism of hyperinsulinemia in PCOS? (2013, 2015, 2017, 2018)
- Increased serine phosphorylation of insulin receptor
- Insulin receptor polymorphism
- Insulin receptor defect
Increased serine phosphorylation of insulin receptor
A patient is on metformin. Which of the following is the best reason to stop it 24hr in advance? (2009, 2011, 2012, 2014, 2015, 2018, 2023 NIH)
- MRI
- HSG
- Intravenous pyelogram (IVP)
- VOR
IVP
Contrast dye can cause renal failure, which results in build up of lactic acidosis (restart metformin 48 hours after dye)
What is least likely to occur if ovarian drilling is performed on a patient with PCOS? (2014, 2015)
- Drop androstenedione
- Decrease dopaminergic inputs
- Decrease LH
- Decrease LH/FSH ratio
Decrease dopaminergic inputs
How does insulin primarily increase androgen production in the ovary? (2010, 2012, 2018)
- Insulin action on IGF-1 receptors of theca cells
- Insulin action on insulin receptors in theca cells
- Insulin action on androgen receptor
- Insulin upregulates CYP17 enzymes
Insulin acts via which of the following to increase androgen production in theca cells? (2023 NIH)
- Insulin receptor
- Stimulation of CYP17 transcription
- IGF-1 receptor
Insulin receptor
What causes hyperinsulinemia with insulin resistance in PCOS? (2018)
- Less insulin receptors
- Intrinsic abnormality in the receptor
- Defects in the post-receptor signaling pathway
Defects in the post-receptor signaling pathway
What is least useful in diagnosis of PCOS? (2007, 2011, 2012, 2018)
- LH:FSH ratio
- Ultrasound (US)
- Ovarian volume
- Serum androgen levels
- Ferriman-Gallwey scores
- Menstrual history
LH:FSH ratio
PCOS patients are at increased risk for which of the following? (2012, 2018)
- Ovarian cancer
- Breast cancer
- Colon cancer
- Renal cancer
Breast cancer
What is the most common abnormality in PCOS? (2014)
- Elevated free testosterone
- Elevated total testosterone
- Elevated DHEA
- LH:FSH ratio >3
Elevated free testosterone
A college student has PCOS. What is the most appropriate therapy? (2011)
- Progestins
- Dexamethasone
- Metformin
- OCPs
OCPs